You are on page 1of 46

COMPLEX COMPOUNDS JEE-ADV CHEM-VOL-II

SR-MAIN-CHEM-VOL-II

COMPLEX COMPOUNDS
SYNOPSIS Ш Secondary valency (Non-Ionisable valency)
i) It is satisfied by anions or neutral molecules or
Co-ordination Compounds rarely with cations. The groups satisfying secondary
Ш Double Salt : valencies are called ligands.
1) Double salts exist only in solid state and dissociate ii) The number of secondary valencies is equal to
into ions in water coordination number.
2) They lose their identity in solution. iii) It is represented by thick lines while writing the
3) The properties of double salt are essentially the structure of the complex.
same as those of the constituent compounds Ш In some complexes the same groups satisfies both
4) In double salts, metal ion exhibit their normal primary and secondary valencies.
valency. Ш The ligands are directed in space around the central
5) Double salts contain ionic bonds metal atom in different ways. This leads to a definite
geometry to the molecule.
Ex: i) KCl . MgCl2 .6 H 2O (Carnallite)
ii) K 2 SO4 . Al2 ( SO4 )3 .24 H 2O (Potash alum) No. of ions
Colour Formula
in solution
iii)FeSO4 . (NH4)2 SO4 . 6H2O (Mohr’s salt)
Ш Coordination compound : Yellow CO NH 3   3Cl 
3
4 ions
 6
1) They exist in the solid state as well as in solution.
2) They donot completely lose their identity in Purple COCl NH 3 5  2Cl 
2
3 ions
solution.
3) The properties of coordinate compounds are COCl2 NH 3 4  Cl 

Green 2 ions
different from the constituent compounds.
4) In coordination compound, metal ion is 
Violet COCl2 NH 3 4  Cl  2 ions
surrounded by a number of anions or neutral
molecules. The metal ion exhibits more than its
normal valency. Complex
No. of
6 Werner structure
ligands
5) Coordinate comounds may have ionic as well as Octahedral
coordinate bonds. –
1. Three Cl ions satisfy primary valency NH3 NH3 NH3

COCl36NH3 6 2. Six NH3 species satisfy secondary valency


Ex: K4[Fe(CN)6], K3[Fe(CN)6], Na3  Ag  S 2O3 2  3. Ions in solution = 4 Cl

CO
+3
Cl

4. AgCl molecules precipitated on adding


Ш The transition metal ions form co-ordinate AgNO3 = 3 AgCl = 3
NH3 NH

3
NH3
Cl
compounds because –
5. Ionisable Cl = 3

i) They have small size. Octahedral



1. Two Cl satisfy primary valency
ii) They have high nuclear charge COCl35NH3 6

2. One Cl satisfy both primary and NH3 NH3 NH3
iii) They have vacant d-orbitals secondary valency
– +3 –
3. 5NH3 satisfy secondary valency Cl CO Cl
iv) They can accept lone pairs of electrons. 4. No of ions in solution = 3
Ш Werner’s Theory: According to Werner 5. AgCl molecules precipitated on adding –
Cl
NH3 NH3
AgNO3 = 2
(father of Co-ordination chemistry) transition –
6. Ionisable Cl = 2
metals possess two types of valencies. 1. Octahedral NH3 NH3
a) Primary valency (Ionisable valency) –
2. 2Cl & 4NH3
3. One Cl satisfies
b) Secondary valency (non ionisable valency) COCl34NH3 6
4. No of species in
Cl

CO
+3
Cl

Ш Primary Valency (Ionisable Valency) 5. AgCl molecules


– NH3 NH3

6. Ionisable Cl = 1 Cl
i) It is referred to as oxidation state. 1. Octahedral NH3 NH3
ii) It is satisfied by only anions. –
2. The three Cl ions and 3NH3 satisfy secondary
iii) It is represented by dotted lines while writing COCl33NH3 6 valency Cl

CO
+3
Cl

3. No. of species in solution = 1


the structure of complex. 4. AgCl molecules precipitated = nil
– NH3

5. Ionisable Cl = 0 Cl

219
219 SR.INTER - IIT ADVANCED - VOL - 2
COMPLEX COMPOUNDS JEE-ADV CHEM-VOL-II
SR-MAIN-CHEM-VOL-II
Defects in Werner’s Theory Definitions of Some Important Terms
Ш This theory does not relate the electronic in co-ordination Compounds
configuration of metal with the formation of the Oxidation number of central atom: The
complex. oxidation number of the central atom in a complex
Ш It is known now that the metal tries to acquire the is defined as the charge it would carry if all the
nearest inert gas configuration during the formation ligands are removed along with the electron pairs
of complex that are shared with the central atom. It is also called
Ш This theory does not explain the reason for the primary valency.
colour of the complex Ш The oxidation number is represented by a Roman
Ш This theory does not explain the magnetic behaviour numerical in parenthesis after the name of the metal.
of complexes.
Coordination number: The coordination
S.E-1:When excess of silver nitrate solution is number (CN) of a metal ion in a complex can be
added to aqueous solution containing 0.1 defined as the number of ligand directly bonded to
metal ion through coordinate bonds (or)  bonds.
molar CoCl3 .xNH 3 . If 43.05 g of silver Ш The co-ordination number is generally 2, 4 (or) 6,
chloride precipitated, then x value is ocassionally 8 (in Osmium complexes).
43.05 Ш It is equal to the number of monodentate ligands
Sol. nAgCl   0.3 which are bound to central metal atom or ion
143.5
through a dative bond. It is also called secondary
0.1 mole of CoCl3 .xNH 3 gives 0.3 mole of AgCl . valency.
Ш Shape of the complex species depends on its co-
1 mole of CoCl3 .xNH 3 gives 3 mole of AgCl
ordination number.
IV is 3. SV of Co is 6 and PV is 3.
3
Co-ordination Shape of the
So, IV = PV, Hence x =6. Number Complexes
IV=Ionisable valency, 2 Linear
PV=primary valency
3 Trigonal planar
Tetrahedral (or)
S.E-2: The secondary valence of Co  3 is 6. 4 square planar
Calculate the number of moles of AgCl
Square pyramidal (or)
5 trigonal bipyramidal
precipitated, when excess of AgNO3 solution
is added to 1.5lit of 0.2M. CoCl3 .5 NH 3 6 Octahedral
solution. Pentagonal
7 bi-pyramidal
Sol. SV of Co  3 is 6 and it will be satisfied by five
NH 3 and one Cl  the remaining 2Cl  ions satisfy Ш Co-ordination Entity or Co-ordination

IV so fomula is CoCl  NH 3 5  Cl2 . sphere: A coordination entity constitutes a central


metal atom or ion bonded to a fixed number of ions
1 mole of this complex gives 2 mole of Cl ions

or molecules. It is enclosed in square bracket and
1.5 x 0.2 mole gives how many moles of Cl  ions. is collectively termed as the coordination sphere.
The ionisable groups are written outside the bracket
1.5  0.2 and are called counter ions
 2  0.6moles
1 Ш In CoCl  NH 3 5  Cl2 complex,

220
SR.INTER - IIT ADVANCED - VOL - 2 220
COMPLEX COMPOUNDS JEE-ADV CHEM-VOL-II
SR-MAIN-CHEM-VOL-II
Ш A molecule or ion which donates two lone pairs of
CoCl  NH 3 5  is coordination sphere and 2Cl 
electrons is called bidentate ligand. The bidentate
ions are counter ions. ligand will have two co-ordinating centers.
Ш Co-ordination entities are mainly two types. They Ex: 1) Oxalate ion 2) Glycinate ion
are 3) Ethylene diamine
a) Mononuclear complexes Ш Bidentate ligand: contains two donor atoms
Ex :- oxalate, ethylenediamine, DMG, carbonate,
Ex: K 4  Fe  CN 6  , CrCl  H 2O 5  Cl2 glycinate
b) Polynuclear complexes Ш Tridentate ligand: contains three donor atoms
Diethylene triamine, iminodiacetate, 2,2’,2’’-
Ex:  Co2  NH 3 6  OH 3  Cl3 terpyride
Ш Mono nuclear complexes are three types. Ш Tetradentate ligand: contains four donor
1) Neutral complexes : atoms. Triethylene tetramine
Ш Pentadentate ligand: contains five donor
complex donot posses charge
atoms. Ethylene diamine triacetate (EDTA)-3
Ex: Co  NH 3 3 Cl3  ;  Pt  NH 3 2 Cl4 
2) Cationic complexes :
Complex posses positive charge
Ex : Cu  NH 3 4  SO4 ; Co  NH 3 6  Cl3
3) Anionic complexes :
Ш Hexadentate ligand: contains six donor atoms.
complex posses negative charge (EDTA)-4 (ethylene diamine tetra acetate)
Ex: K 4  Fe  CN 6  , K 3 CoCl6 

Based on ligands types of complexes:


Ш Homoleptic complex: The complex in which
the central metal atom (or) ion bound with only one
4
kind of ligand.Ex:  Fe  CN 6  , Co  H2O6 
3
Ш Ambidentate Ligand: It contains two donor
atoms but both atoms can’t form dative bond in
Ш Heteroleptic complex: The complex in which single compound.
the central metal atom (or) ion bound with more
2 O
than one kind of ligand. Ex: CO  NH 3 5 Cl 
M N M O N O
Ligand: An ion or a molecule which can donate a O
pair of electrons to a metal atom or a metal ion and nitrito-N nitrito-C
can form dative bond is called ligand. They are Lewis
acids. M SCN M NCS
Ш Ligands are three types thiocyanato isothiocyanato
Negative ligands Eg: Cl-, SO4-2, CN-, C2O4-2; etc.,
EDTA can acts a penta and hexa dentate ligand.
Neutral ligands Eg: H2O, NH3 etc.,
So., it is called felxi dentate ligand.
Positive ligands Eg : NO+, NO2+
Ш A molecule or ion which donates only one pair of Ш Chelating ligands: A multi dentate ligand
electrons is called monodentate ligand. The simultaneously coordinate to metal ion through more
monodentate ligand is said to have only one than one sigma bond, a ring like structure obtained.
co-ordinating center. which is called Chelate and ligand is called Chelating
ligand.
Ex: CN  , HO  , NH 3 , H 2O

221
221 SR.INTER - IIT ADVANCED - VOL - 2
COMPLEX COMPOUNDS JEE-ADV CHEM-VOL-II
SR-MAIN-CHEM-VOL-II
Ш Chelating effect: Due to this stability of complex H 2O - aqua (or) aquo
increases. NO - Nitrosyl
2
Ex : Cu  en  2  is less stable because it forms NH 3 - ammine
1
CS - Thiocarbonyl
only two rings.  Fe  EDTA   is more stable CO - Carbonyl
because it forms five rings. NS - Thionitrosyl
Chelate contains 6 donor atoms are less stable due Ш Suffix for negative ligand is ‘o’.
to steric hinderance. Ex: Cl  - Chloro, OH  - hydroxo
Ш Importance of Chelates: Chelates used in NO2 - Nitrito, CO32 - Carbonato
(i) softening of hard water
(ii) separation of lanthanides and actinides. C2O42 - Oxalato, C6 H 5COO  - Benzoato
(iii) estimation of Ni+2, Mg+2, Cu+2 ions qualitatively. Br  - Bromo, I  - Iodo, F  - Flouro
Ш Central atom/ion: In a coordination entity, the SO42 - Sulphato, O22 - Peroxo,
atom/ion to which a fixed number of ions/groups
are bound in a definite geometrical arrangement H  - Hydrido, NH 2 - amido,
around it, is called the central atom or ion. NH 2 - Imido C2O42 - Oxalato etc..
Ш For example, the central atom/ion in the coordination Ш Suffix for positive ligand is ‘ium’.
entities: [NiCl 2(H2O)4], [CoCl(NH3)5] 2+ and
[Fe(CN)6]3– are Ni2+, Co3+ and Fe3+, respectively. Ex: NO2 - Nitronium, Cl  - Chloronium
These central atoms/ions are Lewis acids. 
NO  - Nitrosonium, N 2 H 5 - Hydrazinium
IUPAC nomenclature Ш Order of Naming Ligands:If more than one
Ш Order of Naming of Ions: The positive ion is different kinds of ligands are present in a compound,
named first followed by the negative ion. their names should be written in alphabetical order.

Ex: K 4  Fe  CN 6   4 K    Fe  CN 6 
4 Ex :  Pt  Br  Cl  NO2  NH 3   is named as
ammine bromo chloro nitro platinate (II) ion.
Potassium hexacyano Ferrate (II)
Ш Numerical prefixes to indicate number of
Ш Name of the non ionic (or) neutral complex must
ligands: If more than one same kind of ligands
be written in one word.
are present they are labled as di, tri, tetra, penta
Ex :  Ni  CO 4  - tetra carbonyl nickel (O). etc. Ex : Cr  NH 3 6  Cl3 -
Ш Naming the Co-ordination Entity: In Hexaamminechromium(III) chloride.
naming the coordination entity, the ligands are named 2
first and then central metal ion.  Ni  CN  4  - Tetracyanonickelate(II)ion.
Ш Naming of ligands : ligands are 3 types Ш If the same ligand can donate lone pairs from more
Ш No characteristics suffix for neutral than one centre, they are named as bidentate,
ligands. tridentate etc, depending on the number of lone
pairs donated.
Ex: NH 2 CH 2 CH 2 NH 2 - Ethylene diammine
Ex: NH 2CH 2CH 2 NH 2 .
C6 H 5 N - Pyridine Ш The number of bidentate, tridentate etc ligands is
PH 3 - Phosphine mentioned with bis, tris, tetrakis, etc. If they already
contain di, tri, tetra etc. in their names.
 C6 H 5 3 P - Triphenylphosphine
222
SR.INTER - IIT ADVANCED - VOL - 2 222
COMPLEX COMPOUNDS JEE-ADV CHEM-VOL-II
SR-MAIN-CHEM-VOL-II

Ex:  Pt Br2  NH 2 CH 2 CH 2 NH 2 2  Cl2 is  NO2 (through N) O NO  (through -O)


named as dibromobis (1, 2-ethane diamine) Nitrito -N (or) Nitrito-O (or)
Platinum(IV) chloride. Simply nitro Simply nitrito
Ш Ending Names: When the complex is anionic,  SCN  (through -S)  NCS  - (through -N)
the name of the central metal atom ends with -ate. thio cyanato (or) isothio cyanato (or)
For cationic and neutral complexes the name of the thio cyanato-S thio cyanato-N
metal is written without any characteristic ending.
Cationic Complexes
Ex: Cationic complex :
Co  NH 3 6  Cl3 -Hexamminecobalt(III) chloride Ш  
Co NH 3 6  Cl3

 
Hexamminecobalt (III) chloride
Ex :Neutral complex :  Ni  CO  4  -  Pt  Cl  NH   Cl
Ш 3 5
Tetracarbonylnickel(0)   3
Pentaamminechloroplatinum (IV) chloride
Ex :Anionic complex : K  Pt Cl5  NH 3   -
Ш CrCl  NO2  NH 3 4  NO3
Potassium amminepentachloroplatinate(IV)
Ш If the complex is anion, the name of the metal should Tetraamminechloronitrochromium(III) nitrate
be taken from latin language.
Ш  Pt  NH 3 6  Cl4
Ex : Cu - Cuprum – Cuprate  
Sn - Stannum – Stannate Hexammineplatinum(IV) chloride.
Fe - Ferrum – Ferrate Ш Co  NH3   H 2O  Cl  Cl2
Pb - Plumbum – Plumbate  4 
Tetraammineaquachlorocobalt(III) chloride.
Ag - Argentum– Argentate

Au - Aurum – Aurate Ш Cr  H 2O  4 Cl2 
Cr - Chromium – Chromate
Ni - Nickel – Nickelate Tetraaquodichlorochromium(III)ion.

Oxidation State of Central Metal ion: Ш  Ag  NH 3 2  Cl Diamminesilver(I) chloride.


The oxidation state of the central metal ion is
designated by a Roman numerical (such as II, III, Ш Ti  H 2O 6  Cl3
 
IV) in the brackets at the end of the name of the Hexaaquotitanium(III) chloride.
metal of the complex. Ex : CoCl  NH 3 5  Cl2 Ш Cr  NH 3 6  Cl3
 
- Pentaamminechlorocobalt(III) chloride. Hexaamminechromium(III) chloride.
Ш Point of Attachment: When a ligand can
Ш  Cu  en  2  SO4
coordinate through more than one atom, then the
point of attachment of the ligand is indicated by Bis(ethylenediamine)copper(II) sulphate.
putting the symbol of the atom through which Ш Cu  NH   SO
coordination occurs, after the name of the ligand.  3 4 4

Sometimes, different names are used for alternative Tetraamminecopper(II) sulphate


modes of attachment. Ш  Fe  H 2O 4  C2O4   2 SO4

Ex: NO2 can co-ordinate through -N (or) -O. If Tetraaquaoxalatoiron(III) sulphate.
it co-ordinates through N, it is called nitrito -N 
Ш  Fe  H 2O 5 NO  SO4
(or simply as nitro). On the other hand, if it co-
ordinates through -O, (-ONO-), it is called nitrito- Pentaaquanitrasonium(I) sulphate
O.
223
223 SR.INTER - IIT ADVANCED - VOL - 2
COMPLEX COMPOUNDS JEE-ADV CHEM-VOL-II
SR-MAIN-CHEM-VOL-II
Anionic Complexes Isomerism in Complexes: Substances
having the same molecular formula but have different
Ш K 4  Fe  CN 6 
structures (or) properties are known as isomers.
Potassium hexacyanoferrate(II) The phenomenon of the existence of isomers is
Ш Na2  Zn Cl4  Sodium tetrachlorozincate(II) known as isomerism. It is two types
1) Structural isomerism 2) Stereo isomerism
Ш K 3  Fe  CN 5 NO  Ш Structural Isomerism: This isomerism arises
Potassium pentacyanonitrosylferrate(II) due to the difference in the structures of complexes.
Ш Ionisation Isomerism: Complexes which have
Ш K 3  Fe  CN 6  the same molecular formula but gives different ions
Potassium hexacyanoferrate(III) in solution are called Ionisation isomers and the
phenomenon is called ionisation isomerism.
Ш K3 Cr  C2O4 3 
Ex: Co  NH 3 5 SO4  Br and Co  NH 3 5 Br  SO4
Potassium trioxalatochromate(III)
Ш Ionisation isomers are formed by the interchange
Ш K 3 CoCl2  C2O4 2  of the position of ligands inside (or) out side the
Potassium dichlorodioxalatocobaltate(III) coordination sphere.
Ш Ionisation isomerism arises, when the counter ions
Ш K2  HgI4  Potassium tetraIodomercurate(II).
in a complex compound can also function as
Ш K 2  Pt Cl6  ligands.
Potassium hexachloroplatinate(IV) Ш In the ionic complex compounds, the ion having an
opposite charge to that of the complex ion is called
Ш Na  Ag  CN 2  counter ion.
Sodium dicyanoargentate(I) i) K in K 4  Fe  CN 6 
2
Ш  Ni  CN  4 
ii) Cl ions in Co  NH 3 6  Cl3
Tetracyanonickelate(II) ion. Ш The following pairs of compounds give different ions
Ш K 3 Cr  CN 6  in solutions.
Potassium hexacyanochromate(III) Ш Co  NO3  NH 3   SO4 gives sulphate ions while
 5

Ш K 3 Co  CN 6  Co  SO4  NH 3 5   NO3  gives nitrate ions.


 
Potassium hexacyanocobaltate(III)
Ш Co  NO3  NH 3 5  SO4 gives white
Na3  Ag  S2O3 2   
Ш
precipitate with BaCl2 solution.
Sodium bis(thio sulphato)argentate(I)
Neutral Complex Ш Co  SO4  NH 3 5   NO3  cannot give white
 
Ш Co  NH 3 3 Cl3 
  precipitate with BaCl2 solution.
Triamminetrichlorocobalt(III)
 Pt  NH 3  Br2Cl2  Ш  Pt Cl2  NH 3 4  Br2 gives Br  ions. So it can
Ш  2 
Diamminedibromodichloroplatinum(IV) give light yellow precipitate with aqueous AgNO3
Ш Cr  H 2O 3 Cl3  .3H 2O solution.
 
Triaquotrichlorochromium(III)trihydrate. Ш  PtBr2  NH 3 4  Cl2 gives Cl  ions. So it can
Ш Fe  CO 5 Pentacarbonyliron(O)
give white precipitate with aqueous AgNO3
Ш Co  NH 3 3  NO2 3  solution.
 
Triamminetrinitrocobalt (III)
224
SR.INTER - IIT ADVANCED - VOL - 2 224
COMPLEX COMPOUNDS JEE-ADV CHEM-VOL-II
SR-MAIN-CHEM-VOL-II

Ш Co  SO4  NH 3 5  Br is red coloured and it 3


   Co  CH 3CH - CH 2  
  | |  
gives Br  ions, where as Co Br  NH 3 5  SO4 is   NH 2 NH 2  3 
violet coloured and it gives sulphate ions.
Ш Stereo Isomerism: Isomers which have the
Ш Hydrate Isomerism: Hydrate isomerism is a same position of atoms (or) groups but differ in the
special type of ionization isomerism. This type of spatial arrangements around the central atom are
isomerism arises due to the presence of water called stereo isomers and the phenomenon is called
molecules in and outside the coordination sphere. stereo isomerism. It is classified into two types.
Ш This type of isomers differ in number of water
(a) Geometrical isomerism
molecules as ligands and as hydrated molecules.
(b) optical isomerism.
Ex : CrCl3 .6 H 2O molecular formula has four
possible isomers. They are Geometrical Isomerism: Geometrical
isomerism is due to ligands occupying different
Cr  H 2O 6  Cl3 - violet coloured; positions around the central metal atom (or) ion.
 
Ш When two identical ligands occupy adjacent
Cr  H 2O 5 Cl  Cl2 .H 2O - light green coloured positions, the isomer is called cis-isomer.
 
Ш When two identical ligands occupy opposite
CrCl2  H 2O 4  Cl.2 H 2O -darkgreen coloured positions, the isomer is called trans-isomer.
Ш Geometrical isomerism is very much common in
CrCl3  H 2O 3  .3H 2O - brown coloured
  coordination number 4 and 6 complexes.
Ш Tetrahedral complexes cannot exhibit geometrical
Ш In the same manner CoCl3  H 2O 6 complex has
isomerism.
the isomers CoCl  H 2O 5  Cl2 .H 2O ; Ш Square planar complexes of the type
CoCl2  H 2O  4  Cl.2 H 2O &CoCl3  H 2O   .3H 2O. Ma4 , Ma3b, Mab3 do not exhibit geometrical
 3
isomerism.
Ш Linkage Isomerism: Complex with
Ш Square planar complexes of the type
ambidentate ligands shows this type of isomerism.
A mono dentate ligand with two (or) more donar Ma2b2 , Ma2bc and Mabcd exhibit geometrical
atoms is called ambidentate ligand. isomerism.
Ex: Co( NO2 )( NH 3 )5 2 & Co(ONO)( NH 3 )5 2 Ex :  Pt  NH 3  2 Cl2 
Ш Co-ordination Isomerism: It arises due to
the mutual exchange of ligands between anionic and
cationic spheres. Cl  Cl 
Ex: Co( NH 3 )6 Cr (CN )6  &

Co(CN )6 Cr ( NH 3 )6 
Cl  Cl 
Ш Ligand Isomerism: This type of isomerism
arises due to the presence of isomers of the ligands
in different isomers of the complex compound.
3
Ex :  Co  H 2 NCH 2 CH 2 CH 2 NH 2 3  &

225
225 SR.INTER - IIT ADVANCED - VOL - 2
COMPLEX COMPOUNDS JEE-ADV CHEM-VOL-II
SR-MAIN-CHEM-VOL-II

L4 L1 L1

L5 L2
M
Ш M

L3 L2
L4 L3
Cis Positions : 1,2; 2,3; 3,4; 1.4
L6
Trans Positions : 1,3; 2,4
Cis Positions : 1,2; 1,3; 1,4; 1,5; 2,3; 2,5;
Ш Mabcd type of complexes will have 3 geometrical 3,4; 4,5; 2,6; 3,6; 4,6; 5,6
isomers i.e. 2cis and 1 trans isomer Trans Positions: 1,6; 2,4; 3,5
Ш Square planer complex having unsymmetrical Ш octahedral complexes of the type Ma , Ma b ,
6 5
bidentate chelating ligands with general
n Mab5 do not exhibit geometrical isomerism.
formula  M  AB  2  exhibit geometrical
Ш Octahedral complexes of the type
isomerism Ma b , Ma bc and Ma b complexes exhibit
4 2 4 3 3
geometrical isomerism.

Ex : CoCl2  NH 3 4 

Ex: NH3 Cl–

NH3 Cl– H3N NH3

Ш Square planer complex having symmetrical bidentate


CO3+ CO3+
ligands with general formula
n
 M  AA  2  can also exhibit cis-trans isomerism NH3 –
H3N NH3
Cl
2
NH3 Cl–
Ex:  Pt II  NH 2 CH  CH 3  CH  CH 3  NH 2 2 
Cis-isomer Trans-isomer

CH3 CH3 Ш Ma3b3 type of complexes exhibits maridian and


C – NH2 NH2– C facial type of isomers
H H and
Pt
CH3 CH3 b
C – NH2 NH2– C
H CH3 a b
H CH3
C – NH2 NH2– C M
CH3 H
Pt
H CH3
C – NH2 NH2– C a b
CH3 H

Ш Octahedral complexes
a
facial

226
SR.INTER - IIT ADVANCED - VOL - 2 226
COMPLEX COMPOUNDS JEE-ADV CHEM-VOL-II
SR-MAIN-CHEM-VOL-II
optically inactive.
b Ш Optical Isomers have almost identical physical and
chemical properties.
a a Ш The optical isomers can be distinguished only by
their rotation of the plane polarised light.
Ш Optical isomers are also generally provided by the
M
octahedral coordination compounds having
bidentate (or) polydentate ligands.
a b
 3
Ex: CoCl2  en  2  , Cr  C2O4 3  ,
b
 2
Ш Octahedral complexes with general formula Cr  NH 3  Cl2 en  ;  Pt Cl2  en 2 
 2 
n n
 M  AA2 a2  ,  M  AA  2 ab  , Here ' en ' represents H 2 NCH 2CH 2 NH 2
n Ш Optical isomerism is shown by the complexes having
 M  AA  a2b2  can exhibit geometrical chiral structures. They rotate the plane of polarised
isomerism. light in opposite directions.
Ш Octahedral complexex containing unsymmetrical a) Complexes with C.N. = 4
bidentate chelating ligands with general formula Ш Square planar complexes do not show optical
n isomerism.
 M  AB 3  can exhibit geometrical isomerism.
Ш Tetrahedral complexes with formula M  AB 2
Ш Optical Isomerism: The compounds having
can exhibit optical isomerism.
same molecular formula but differ in the rotation of
Ш Octahedral complexes with general formula
plane polarised light are called optical isomers and
the phenomenon is called optical isomerism.  Ma2b2 c2  n ,  Ma2b2 cd  n ,  Ma2bcde n and
Ш The isomer which rotates the plane polarised light n
to the right direction is called dextro (d)-form while  Mabcdef  can exhibit optical isomerism.
the isomer which rotates the plane polarised light Ш Octahedral complexes containing symmetrical
to left direction is called laevo ( l )-form. bidentate ligands with general formula
Ш Co-ordination number four, tetrahedral complexes n n
 M  AA 3  ,  M  AA  2  BB   can exhibit
of the type ( M abcd ) exhibit optical isomerism.
optical isomerism.
Ex :  Pt  NH 3  H 2O  Cl  NO2  
Py 2+ Py 2+
Cl Py Py Cl
NH3 NH3
Pt Pt
Cl NH3 Cl
H3N
Pt Pt NH3 NH3
Mirror
NO2 NO2 H2O cis-d-isomer cis-l-isomer
H 2O
Cl Cl Ш Octahedral complexes containing unsymmetrical
d-form Mirror l-form bidentate ligands with formula
Ш A pair of optical isomers which are non super n
imposable mirror images are called enantiomers or  M  AB 3  ( both Cis & Trans ) can exhibit
enantiomorphs. optical isomerism.
Ш An equimolar mixture of d-forms and l-forms is Ш Octahedral complexes containing symmetrical
called racemic mixture. Racemic mixtures are
227
227 SR.INTER - IIT ADVANCED - VOL - 2
COMPLEX COMPOUNDS JEE-ADV CHEM-VOL-II
SR-MAIN-CHEM-VOL-II
bidentate and monodentate ligands with general 2
dsp 2 Square planar  Ni  CN 4 
n
formula Cis   M  AA 2 a2  ,
n
3d x2  y 2
4 s 4 pz 4 p y   Ni  NH 3 4 
2

Cis   M  AA 2 ab  , Cu  NH 3 4 


2
CN  4
n 3
Cis   M  AA  a2b2  , and dsp 3 Trigonal CuCl5 

Cis   M  AA  BB  a2 
n
are optically active.
 3d z2
4s 4 p3  bipyramidal  Fe  CO 5 
0

CN  5
Ш Octahedral complexes containing polydentate 2
sp 3 d Square  SbF5  , IF5
ligands are optically active.

Ex: Co III


 EDTA 

.
 4s, 4 p 4d  pyramidal
3
x2  y 2

CN  5
3
S.E-3: The number of stereo isomers of the given d 2 sp 3 Inner orbital Cr  NH 3 6 

compound is Co  C2O4   NH3 2  NO2 2  . 3d x2  y 2 
3d z 2 4s 4 p 3 octahedral  Mn  CN 6 
3

3
Sol. Four CN  6  Fe  CN 6  etc
VBT of Co-ordination Compounds: 3
3 2
Ш The central metal atom or ion makes available a sp d Outer orbital  FeF6 
number of empty s, p and d atomic orbitals equal 3

to its coordination number. These vacant orbitals 4s 4 p 4d x2  y2 4d z 2 octahedral 
Co  H 2O 6 
2

hybridise together to form hybrid orbitals. These 2


hybrid orbitals are vacant, equivalent in energy and CN  6  Ni  NH 3 6 
have definite geometry. The most common Ш Each ligands have atleast one sigma orbital
hybridizations in complex are given. containing alone pair of electrons.
Type of Geometry of Examples
Ш Vacant hybrid orbitals of the central metal atom or
hybridization complex

ion overlap with the filled (containing lone pair of
sp Linear or  Ag  NH 3 2  , electrons) sigma orbitals of the ligands to form metal-
 ligand sigma bond. This bond is coordinate bond.
 4s4 p  diagonal  Ag  CN 2 
Ш If d orbitals are involved in the hybridisation than
CN  2 that may be either inner (i.e., (n-1) d-orbital) or the
 outer (i.e., nd-orbital).
sp 2 Trigonal planar  HgI 3 
Ш If (n - 1)d orbitals are used for hybridization along
 4 s 4 p 2
 with ns and np orbitals, such complex is called inner
orbital complex. If nd orbitals are used for
CN  3 hybridisation along with ns and np orbitals then it is
0
sp 3
Tetrahedral  Ni  CO 4  called outer orbital complex.
2
Ш Some times the unpaired (n-1) d-electrons
 4s4 p3   Zn  NH 3 4  undergoes rearrangement and provides more
2 number empty orbitals, which is possible in presence
CN  4  ZnCl4  of strong ligands such as
2
CuX 4  CO, CN  , NO2  , en, NH 3 , Py and EDTA.
2
 MnX 4  Ш The non-bonding electrons of central metal atom
2 or ion remain unaffected and do not take part in
 NiX 4  chemical bonding.
X  Cl  , Br  , I  Ш During complex formation, the Hund’s rule of
228
SR.INTER - IIT ADVANCED - VOL - 2 228
COMPLEX COMPOUNDS JEE-ADV CHEM-VOL-II
SR-MAIN-CHEM-VOL-II
maximum multipliciy is strictly followed. However, 3d5
under the influence of strong ligands, the electrons ×× ×× ×× ×× ×× ×× ××
may be forced to pair up against Hund’s rule (but
only for (n-1)d orbits) 3 2
sp d -hybridisation
Ш Complex containing unpaired electron (unpaired Geometry : Octahedral
electron of central metal atom or ion) is paramagnetic No. of unpaired electrons/paramagnetic
in nature. If all the electrons of metal atom or ion character: 5-Paramagnetic.   5.86BM
are paired, the complex is diamagnetic in nature.
4
Inner orbital complex, low spin complex, spin 4. Complex/ion :  Fe  CN 6  Fe2 
paired complex or covalent complex
Electronic configuration and hybridisation:
Hyperligated complex
Outer orbital complex, high spin complex, 3d6
spin free complex, or ionic complex, Hypo ×× ×× ×× ×× ×× ××
ligated complex
Ш The spin only magnetic moment of the complex can
d2sp3-hybridisation
be calculated by the formula S  n  n  2  BM Geometry : Octahedral
( n = number of unpaired electrons ) No. of unpaired electrons/paramagnetic
Examples of Complexes character: 0-Diamagnetic
4 2
1. Complex/ion :  Mn  CN 6  5. Complex/ion :  Fe  H 2O 6  Fe 2 
Electronic configuration and hybridisation of Mn 2 : Electronic configuration and hybridisation:
5
3d 4s 4p 3d6
×× ×× ×× ×× ×× ××
×× ×× ×× ×× ×× ××
sp3d2-hybridisation
2 3
d sp -hybridisation
Geometry : Octahedral
Geometry : Octahedral
No. of unpaired electrons/paramagnetic
No.of unpaired electrons/paramagnetic
character: 4-Paramagnetic.   4.85BM
character: 1-Paramagnetic.   1.76BM
3
2. Complex/ion :  Fe  CN 6  Fe3
3
6. Complex/ion : Co  NH 3 6  Co 3
Electronic configuration and hybridisation: Electronic configuration and hybridisation:
3d5 3d6 4s 4p
×× ×× ×× ×× ×× ×× ×× ×× ×× ×× ×× ××

d2sp3-hybridisation 2 3
d sp -hybridisation
Geometry : Octahedral Geometry : Octahedral
No. of unpaired electrons/paramagnetic
No. of unpaired electrons/paramagnetic
character: 1-Paramagnetic.   1.76BM character: 0-Diamagnetic
3
3. Complex/ion :  Fe  H 2O 6  Fe3 7. Complex/ion: CoF6  Co3
3

Electronic configuration and hybridisation:


Electronic configuration and hybridisation:

229
229 SR.INTER - IIT ADVANCED - VOL - 2
COMPLEX COMPOUNDS JEE-ADV CHEM-VOL-II
SR-MAIN-CHEM-VOL-II
3d6   2.8BM
×× ×× ×× ×× ×× ×× 12. Complex/ion :  Ni  CO 4  Ni 0
Electronic configuration and hybridisation:
sp3d2-hybridisation
Geometry : Octahedral
No. of unpaired electrons/paramagnetic
character: 4-Paramagnetic.   4.85BM
2
Geometry : Tetrahedral
8. Complex/ion: CuCl4  Cu 2  No. of unpaired electrons/paramagnetic
character: 0-Diamagnetic
Electronic configuration and hybridisation: 2
13. Complex/ion :  Pt  CN 4  Pt 2
Electronic configuration and hybridisation:

Geometry : Tetrahedral
No. of unpaired electrons/paramagnetic Geometry : Square Planar
character:1-Paramagnetic.   1.75BM No. of unpaired electrons/paramagnetic
character: 0-Diamagnetic
2
9. Complex/ion :  Zn  NH 3 4  Zn 2  Limitations of VBT
Electronic configuration and hybridisation:
Ш It gives only the qualitative explanations for
complexes.
Ш It does not explain the detailed magnetic properties
of complexes.
Ш This thoery does not explain the spectral properties
Geometry : Tetrahedral of coordination compounds.
No. of unpaired electrons/paramagnetic Ш It does not explain the thermodynamic and kinetic
character: 0-Diamagnetic stabilities of different coordination compounds.
2
10. Complex/ion :  Ni  CN 4  Ni 2 Ш It does not distinguish between weak and strong
ligands.
Electronic configuration and hybridisation:
Ш It does not make exact predictions regarding
terahedral or square planar coordinations entities
with co-ordination number is 4.

S.E-3: The spin only magnetic moment of  FeBr4 
Geometry : Square Planar is 5.92 BM. Predict the geometry of complex
No. of unpaired electrons/paramagnetic ion.
character: 0-Diamagnetic Sol. Secondary valence of Fe 3 ion is 4. So, complex
2
11. Complex/ion :  NiCl4  Ni 2
is square planer  dsp  or tetrahedral  sp  . Br
2 3
Electronic configuration and hybridisation:
is weak field ligand so, rearrangement of d-electrons
is not possible. Hence complex is tetrahedral ( sp3 -
hybridization).
2
Geometry : Tetrahedral S.E-4:  Ni  CN 4  is inner complex but
No.of unpaired electrons/paramagnetic 2
character:  NiCl4  is outer complex. Give the reason.
2-Paramagnetic
230
SR.INTER - IIT ADVANCED - VOL - 2 230
COMPLEX COMPOUNDS JEE-ADV CHEM-VOL-II
SR-MAIN-CHEM-VOL-II
2
Sol. CN  is strong field ligand. So. in  Ni  CN 4  EAN of Pt  78  4  2  6  86  Rn 
Ш In some complexes, the EAN is not equal to the
ion Ni 2 configuration is atomic number of the nearest inert gas. Yet they
8
3d
Hence hybridisation is dsp 2 (square planar). But are stable.
Cl is weak field ligand. So, rearrangement of d- 2
Ex: K 3  Fe  CN 6  ,  Ni  CN 4  .
electrons is not possible. Hence the configuration
y
of Ni 2 is then hybridisation S.E-5: If  AuCl6  follows EAN rule, then the
3d8
is sp 3 (tetrahedral). oxidation state of Au is (At. no. of Au =79
and At. no. of Rn =86).
Effective atomic number (EAN) Sol. EAN = Z - x + n x y
Ш Effective atomic number (EAN) concept was
introduced by Sidgwick to explain the stability of 86 = 79 - x + 6 x 2 ; x = +5
complexes.
Ш The resultant number of electrons with the metal S.E-6: If Ni  CO  follows EAN rule, then x - value
x
atom or ion after gaining electrons from the donar
atoms of the ligands in a complex is called effective is
atomic number (OR) Sol. EAN = Z - x + n x y
The total number of electrons present around central 36 = 28 - 0 + n x 2 ; n = 4.
metal ion in a complex is called effective atomic
number. Formula of compound is  Ni  CO  4  .
Ш Sidgwick proposed that complex ion is stable. If
EAN is equal to the atomic number of the nearest Crystal Field Theory (CFT)
noble gas element. Ш This theory is based on theoretical work of Bethe
& Van Vleck on interaction of ionic crystals.
Ш EAN of a central metal can be calculated by the
Ш Ligands are either anion or neutral molecule
following formula. containing atleast one lone pair of electron. The
Ш EAN  Z  x  n  y anions are regarded as negative point charges. The
x = charge of metal ions neutral ligands are regarded as point dipoles.
n = co-ordination number ; y =2 Ш The neutral ligands are polarised by positive charges

Ex. (1)In K 4  Fe  CN 6  complex of metal ion.


Ш Central metal ion is surrounded by anions or neutral
EAN of Fe  26  2  6  2  36 .
ligands.
3
2)In Co  NH 3 6  complex ion +
+
EAN of Co  27  3  2  6  36 – – –

3)In  Ni  CO 4  complex – M+ – M+

EAN of Ni  28  0  2  4  36 –
– – +
+ –
2
4)In  Ni  CN  4  complex ion
+
EAN of Ni  28  2  2  4  34
3 Ш The electrostatic attraction between nucleus of
5)In  Fe  CN 6  complex ion cation and negative charge of ligands.
Ш Repulsive forces arise between lp of ligands,
EAN of Fe  26  3  2  6  35
electrons of d-orbitals of central metal atom.
6)In K 2  Pt Cl6  complex Ш In a free metal atom, all five orbitals of d-sub shell
have same energy. They are called degenerated
231
231 SR.INTER - IIT ADVANCED - VOL - 2
COMPLEX COMPOUNDS JEE-ADV CHEM-VOL-II
SR-MAIN-CHEM-VOL-II
orbitals. distribution.
When ligands approach the central metal atoms, 1) If  0  p, the fourth electron enters one of the
the electrons of d-orbitals of metal suffer repulsions
by lp of ligands as a result digeneracy is broken eg orbital giving the configuration t23g e1g . Ligands
and d-orbitals are split into two sets of orbitals. for which  0  p are known as weak field ligands
Ш a) Three of d-orbitals i.e., d xy , d yz and d zx which and form high spin complexes.
are oriented in between the co-ordinate axes are 2) If  0  p , it becomes more energetically
called t2g-orbitals.
favourable for fourth electron to occupy a t2 g orbital
Ш b) The other two d-orbitals i.e., d x2  y 2 and d z 2 4 0
which are oriented along the axes are called eg- with configuration t2g eg . Ligands which produce
orbitals. this effect are known as strong field ligands and
Ш This spliting of d-orbitals of metal ion under the form low spin complex.
influence of approaching ligands is called crystal
field spliting. It is designated by  and is called Crystal Field Splitting in octahedral
crystal field splitting energy. and tetrahedral complexes
Ш The decreasing order of field strength among some Ш In octahedral complexes, the six ligands approach
of the ligands are the central metal ion along the co-ordinate axes i.e.,
Weak field ligand the axes of d x2  y 2 and d z 2 orbitals. Consequently,,
    2 
I  Br  SCN  Cl  S  NO3
the eg set of orbitals has higher energy than t2 g
 F   OH   ox 2  O 2  H 2O orbitals.
Strong Field ligands Ш In tetrahedral complex, four ligands may be imagined
to occupy the alternate corners of the cube and the
NCS   Edta 4  py  NH 3  en metal ion at the centre of the cube. In this situation,
 dipy  o  phen  NO2   CN   CO the t2 g set of orbital lie relatively nearer to the
For d ions, two possible patterns of electron
4
approaching ligands and therefore t set of d-
2g

weak

232
SR.INTER - IIT ADVANCED - VOL - 2 232
COMPLEX COMPOUNDS JEE-ADV CHEM-VOL-II
SR-MAIN-CHEM-VOL-II

orbitals have higher energy than eg set of orbitals. greater the value of  .
Ш The metal ion with higher oxidation state causes
Ш Relationship between  t and  0 is given as ;
larger  than is done by the ion with lower oxidation
4 state.
t  o
9 Ш  for the entity [Co(H2O)6]3+ will be more than 
Ш While entering electrons into d-orbitals after splitting for the entity [Co(H2O)6]2+
in ligand field, Hunds and pauli’s principle to be
followed. because oxidation state of cobalt is +3 in the first
S.E-7:Arrange the following in the increasing order and +2 in the second.
of stabilization energy of following in presence Nature of the metal ion:
of strong field ligands. Ш For the analogous entities within a group,  values
2
Sol. d  2  0.4  0  0.6  0.8 0 differ. The  value for similar complexes in the same
d 3  3  0.4  0  0.6  1.2  0 oxidation state increases by 30-50% on going from
3d series (first transition series) to 4d series (second
d 4  4  0.4  0  0.6  1.6  0 transition series). This increase is almost of the same
5
d  5  0.4  0  0.6  2 0 amount (30-50%) on going from 4d-series (second
transition series) to 5d-series (third transition series).
d 9  6  0.4  3  0.6  0.6 0 As a consequence of this, coordination entities of
10
d  6  0.4  4  0.6  0 0 second and third transition series have a greater
tendency to form low spin complexes as compared
d 10  d 9  d 2  d 4  d 5
to the first transition series.
S.E-8: Give the stabilization energy of d 7 Geometry of the coordination entity
Ш The  value for tetrahedral coordination entities is
electrons in presence of strong and weak field
about half the  value for octahedral entities i.e.
ligands.
Sol. Strong field ligands 4
t  0
SE  6  0.4  1  0.6  1.8 0 9
Colouring Coordination Compounds
Weak field ligands Ш most of the transition metal complexes are coloured
SE  5  0.4  2  0.6  0.8 0 in their solid or solution form. The transition metals
have the property to absorb certain radiations from
Factors Determining the Magnitude
the visible region of the spectrum and as a result,
of the Orbital Splitting Energy  : The the transmitted or reflected light is coloured.
following factors influence the magnitude Ш In the case of transition metal complexes,the energy
of  . difference between two sets of d-orbitals is very
Nature of ligand: small. When visible light falls on them, the electron
gets raised from lower set of orbitals to higher set
Ш The  value depends upon the nature of the ligand.
of orbitals.
Ш The CFSE depends on the tendency of ligand to Ш In case of octahedral complexes the electron goes
interact with the central metal ion.
from set of d xy , d yz , d zx to set of d x2  y 2 and d 2
z
Ш The ligands which cause only small  value are
orbitals.As a result of absorption of some selected
called weak field ligands while those which cause
wavelength of visible light corresponding to energy
a large  value are called strong field ligands. difference between these sets of energy levels, the
Ш The ligands can be arranged in the increasing field transmitted light gives colour to complexes,
strength in spectrochemical series. 3
Ш For example, the complex Ti  H 2O 6  is purple.
Oxidation state of the metal ion: Ш In this complex, the metal ion has d1 configuration.
Ш Higher the ionic charge on the central metal ion, the
The electron in t2 g orbital is excited to one of the

233
233 SR.INTER - IIT ADVANCED - VOL - 2
COMPLEX COMPOUNDS JEE-ADV CHEM-VOL-II
SR-MAIN-CHEM-VOL-II

 
eg d x2  y2 or d z 2 orbital by absorbing light equal
Wave length Colour of
Colour of
Coordination coordination
to  0 . of light light
entity entity
absorbed(nm) absorbed (transmitted)
This may be expressed as t2 g1eg 0  t2 g 0eg1
Ш The energy corresponding to this transition [CO(CN)6]3– 310 Violet Pale yellow
corresponds to green and yellow lights which are
absorbed from the white light, while the blue and [CO(H2O)6]3+ 475 Blue Yellow orange
red portions are emitted. the solution of comlpex
[CO(H2O)
3 500 Blue green Red
Ti  H 2O 6  , therefore, looks purple. (NH3)5]3+

Ш In octahedral complexes  vary from one metal [Ti(H2O)6]3+ 510 Blue green Purple
ion to another and the nature of the ligands.
Ш Different complexes absorb different amounts of [COCl(NH ) ] 53.5
3 5
4+
Yellow Violet
energies from visible region and exhibit different
colours. [Cu(H2O)6]3+ 600 Red Blue
3+
Ш For example, three complexes of Co as
[Co(H2O)6]3+ ,[Co(NH3)6]3+ and [Co(CN)6]3-. Ш It may be noted that in the absence of ligands, the
According to spectrochemical series, the crystal field crystal field splitting does not occur and hence the
splitting energies will be in the order of ligands as substance is colourless. For example, removal of

H 2O  NH 3  CN water from [Ti(H2O)6]Cl3, on heating makes it
colourless. Similarly, anhydrous CuSO4 is white but
3+ 3+ 3+ CuSO4.5H2O is blue in colour.
[CO(H2O)6] [CO(NH3)6] [CO(CN)6]
Ш The effect of the ligand on the colour of a complex
0 value small Intermediate Large may be illustrated by taking the example of
Excitation [Ni(H2O)6]2+. This is formed when nickel chloride
small Intermediate Large dissolved in water. The aqueous solution of the
energy(
Absorption
complex has green colour. If the bidentate ligand,
wavelength large Intermediate small ethane-1,2-diamine (en) is progressively added in
( the molar ratio of en : Ni as 1 : 1, 2 : 1 and 3 : 1, the
Colour following series of reactions occur resulting different
Orange Blue Violet
absorbed colour changes as :
2 2
Colour Yellow  Ni  H 2O 6   aq   en  aq    Ni  H 2O 4 en   2 H 2O
Blue Yellow
transmitted orange
Pale blue
2 2
Ni  H2O4 en  aq  en aq  Ni  H2O2  en2   aq  2H2O
blue / purple
2 2
Ni  H2O2  en2   aq  en aq Ni  en3   aq  2H2O
Violet
These colour changes are observed due to the
presence of different ligands in the complexes.
Rules for Writing the Formulae of
Co-ordination Compounds
Ш All mononuclear complex entities contain a single
central metal atom, which is symbolised first.

234
SR.INTER - IIT ADVANCED - VOL - 2 234
COMPLEX COMPOUNDS JEE-ADV CHEM-VOL-II
SR-MAIN-CHEM-VOL-II
Ш If more than one ligand of each type is present, CO CO
they represented in the following order. OC
negative, neutral, positive Ni Fe CO
Ex:-
2 OC CO OC
in correct formula :  Fe  NO 
  CO  3
Cl  CO CO
Ni(CO)4 Fe(CO)5
 2
Correct formula :  FeCl  CO 3 NO  . Tetrahedral Trigonal bipyramidal

Ш Complete co-ordination entity is enclosed in square


CO
brackets. Different species present in the formula CO CO CO CO
are written continuously without leaving space. CO CO
Cr
Ш Charge of anionic or cationic complex is indicated OC Mn Mn CO
outside the square brackets as a right superscript. OC CO
CO CO CO
3 CO CO
S.E-9:In Ti  H 2O 6  complex Ti has only one
3
Cr(CO)6
[Mn(CO)10]
electron in 3d-orbital. Its spectrum shows a Octahedral

single absorption peak of 5000 A0 then


corresponding energy is
N 0 .hc
Sol. E 

6.023 1023  6.63  1034  3  108
=
5  107
= 239 kJ / mol
Bonding in Metal Carbonyls: The bonding
Metal Carbonyls in metal carbonyls is described by the following
steps:
Ш Class of coordination compound known as metal (i) There is a donation of lone pair of electrons of
carbonyls in which carbon monoxide (CO) acts carbon (of CO) into the suitable empty orbital of
as ligand. the metal atom. This is a dative overlap and forms
a sigma M  C bond.
Ш These are also called homoleptic carbonyls
(ii) There is a  -overlap involving donation of
(compounds containing carbonyl ligands only). electrons from filled metal d-orbitals into vacant
Ш These compounds contain both  and  character.. anti-bonding  * molecular orbitals of CO. This
results into the formation of M  C  bond. This
Structure of metal carbonyls is also called back donation or back bonding.The
Ш Tetracarbonyl nickel (0) is tetrahedral. metal to ligand bonding creates a synergic effect
Ш Pentacarbonyl iron (0) is trigonal bipyramidal. which strengthens the bond between CO and the
Ш Hexacarbonyl chromium (0) is octahedral. metal.
Ш Some carbonyls have metal-metal bonds.
Ш Decacarbonyl dimanganese (0) is made up of two Stability of Coordination
square pyramidal Mn(CO)5 units joined by
Mn-Mn bond. Ш Compounds in Solution: The stability of a
Ш Octacarbonyl dicobalt (0) has a Co-Co bond complex in solution refers to the degree of
bridged by two CO-CO groups. association between the two species involved in the
state of equilibrium.
Ш The magnitude of the (stability or formation)
equilibrium constant for the association,
quantitatively expresses the stability.
235
235 SR.INTER - IIT ADVANCED - VOL - 2
COMPLEX COMPOUNDS JEE-ADV CHEM-VOL-II
SR-MAIN-CHEM-VOL-II
Ш we have a reaction of the type: Ш Fe(III) ion in group III is identified as
[Fe(NCS)6]3- ion which has blood red colour.
M  4 L  ML4 then the larger the stability
Ш In Photography the developing of negative is based
constant, the higher the proportion of ML4 that exists on complex formation.
in solution. Free metal ions rarely exist in the solution
so that M will usually be surrounded by solvent AgBr  2 Na2 S 2O3  Na3  Ag  S 2O3 2   NaBr
molecules which will compete with the ligand Ш The extraction of silver, gold from their ores makes
molecules, L, and be successively replaced by them. use of a complex compound formed by leaching
we generally ignore these solvent molecules and their ores with NaCN solution.
write four stability constants as follows:
Ш In the electroplating of Ag, large amounts of silver
M + L  ML K1=[ML]/[M][L] are held as the complex, K[Ag(CN)2].
Ш Haemoglobin is a complex which contains Fe+2 ions.
ML+L  ML2 K2=[ML2]/[ML][L] Ш Chlorophyll is a complex which contains Mg+2 ions.
Ш Vitamin - B12 is a complex which contains Co3+
ML2+L  ML3 K3=[ML3]/[ML2][L] ions.

ML3+L  ML4 K4=[ML4]/[ML3][L] ADDITIONAL TOPICS


where K1, K2, etc., are referred to as stepwise Ш Based on stability complexes are two types
stability constants. Perfect or penetrating complexes These are
the compounds in which complex ion is fairly stable
Alternatively, we can write the overall stability
and is either not dissociated or feebly dissociated
constant thus:
in solution. e.g.,
M + 4L  ML4  4 = [ML4]/[M][L]4
4
Ш The stepwise and overall stability constant are K 4  Fe  CN 6   4 K    Fe  CN 6 
therefore related as follows: 
 4 = K1 Ч K2 Ч K3 Ч K4 or more generally,, Ex : Fe 2   6CN 

 n = K1 Ч K2 Ч K3 Ч K4 ....... Kn  feebly dissociated 


2
S.E-10: Zn2  2 NH 3   Zn  NH 3 2  ; K1  2 103 Imperfect or normal complexes: These are
the compounds in which complex ion is less stable
2
 Zn  NH 3 2   2 NH 3  K 2  1.5  10 3 and is reversibly dissociated to give enough simple
ions.
Find out the instability constant ?
2
Sol. K C  K1.K 2  2  103 1.5  10 3  3  106 K 2  Cd  CN  4   2 K    Cd  CN  4 
Instability constant is inverse of stability constant 
Ex : Cd 2   4CN 
1 1
 6
 3.3  105  appreciably dissociated 
K C 3  10
Application of Complexes: CuSO4 Based on reactivity complex are two types
Ш Labile complexes : Those complexes are highly
solution, when mixed with aqueous ammonia, gives
reactive, i.e., in which ligands can be readily
a deep blue complex soluble in water. This is used
replaced by other ligands are called labile
in the identification of Cu 2 ion. It is due to the complexes.
2
formation of the complex Cu  NH 3  4  Ш Inert complexes : Those complexes are less
reactive, i.e., in which the ligands cannot be readily
Ш Ammonium salts are identified by Nessler’s reagent.
It is a complex K 2  HgI 4  -a colourless solution.
236
SR.INTER - IIT ADVANCED - VOL - 2 236
COMPLEX COMPOUNDS JEE-ADV CHEM-VOL-II
SR-MAIN-CHEM-VOL-II
replaced by other ligands are called inert complexes. iii) Vacant d orbitals
Eighteen electron rule The correct statements is / are:
1) i only 2) ii only 3) i & ii 4) i,ii,iii
Sidgwick accounted the effective atomic number
4. The ionizable valency of Ni in Ni(CO)4 is
rule to explain the bonding in transition metal
1) 2 2) 4 3) 0 4) 1
complexes by extending the octet theory of lewis.
5. According to Werner’s theory transition metals
Stability was assumed to be attendant to a noble possesses
gas configuration for the metal. An alternate and 1) only one type of valency
more general statement is that when the metal 2) two types of valencies
achieves its outer shell configuration 3) three types of valencies
of  n  1 d 10 ns 2 np 6 there will be 18 electrons in the 4) four types of valencies
6. The primary valency of the metal ion is
valence orbitals and a closed stable cofiguration.
satisfied by
M – L (metal lignad) adducts closely follow rules
1) neutral molecules 2) positive ions
relating structure to number of valence electrons. 3) negative ions 4) all the above
3
Example: Cr  NH 3 6   valence electrons 7. No of ionizable & non-ionizable Cl ions in
CoCl3 5 NH 3 representively are
3  Cr   12  NH 3   15 electrons
1) 3, 0 2) 2, 1 3) 1, 2 4) 0, 3
Cr  CO 6   valence electrons 8. Central metal ion in complex compound acts
as
6  Cr   12  CO   18 1) Lewis acid 2) Lewis base
Tetragonal distortions – John Teller effect 3) Arrhenius acid 4) Arrhenius base
If the lignads along z – axis in an octahedral complex 9. Which one of the following acts as a Lewis
base in complexes
are moved cither toward or away from the metal
ion, the resulting complex is said to be tetragonally 1) CO2 2) BF3 3) NH3 4) BCl3
distorted. Since there is loss of bonding energy these DEFINITION OF
distortions are not favoured normally. Jahn Teller
CO-ORDINATION COMPOUNDS
effect in certain situation favours such distortion.
10. Potassium ferrocyanide is an example for
The John – Teller theory sates that for a nonlinear 1) Complex salts 2) Normal salts
molecule that is an electronically degenerate state 3) Double salts 4) Basic salts
distortion must occur to lower the symmetry, 11. Example for a coordination compound is
remove the degeneracy and lower the energy. 1) KCl . Mg Cl2 . 6H2O
2) K2 SO4.Al2(SO3).24 H2O
CONCEPTUAL QUESTIONS 3) CoCl3 . 6N H3
4) Fe SO4 . (NH4)2 SO4 . 6 H2O
WERNER’S THEORY 12. In which of the following transition metal
1. The following does not give a precipitate either complexes does the metal exhibit zero
with AgNO3 or BaCl2 oxidation state
1) [Co (NH3)5 Cl] SO4 2) [Co (NH3)3 Cl3] 1) [Co (NH3)6] Cl3 2) [Fe (H2O)6] SO4
3) [Co (NH3)4 Cl2]Cl 4) Co NH 3 2 Cl4 Cl 3) [Ni (CO)4] 4) [Fe (H2O)6] Cl3
2. Which of the following has highest molar 13. The number of moles of ions produced when
conductivity? one mole of Potassium Ferricyanide is
1) [Co (NH3)6] Cl3 2) [Co (NH3)5Cl] Cl2 dissolved in water is
3) [Co (NH3)4 Cl2] Cl 4) [Co (NH3)3 Cl3] 1) 2 2) 4 3) 5 4) 6
3. Transition elements form complexes readily 14. Metal-Isothiocyanato is indicated by
because 1) M-SCN 2)M-NCS 3)M-CNS 4)M-CSN
i) Small size of cation
ii) Large ionic Charge
237
237 SR.INTER - IIT ADVANCED - VOL - 2
COMPLEX COMPOUNDS JEE-ADV CHEM-VOL-II
SR-MAIN-CHEM-VOL-II
15. Number of chlorides satisifying secondary 26. The effective atomic number of central metal
valency in CoCl3.4NH3 ion is wrongly calculated in the following
complex?
1) 2 2) 3 3) 4 4) 1
16. Which of the following is cationic complex 1. In  Ni  CO 4  the EAN of Ni is 36
1) K 4  Fe  CN 6  2)  Ni  CO  4 
2. In K 2  Ni  CN 4  the EAN of Ni is 36
3)  Co  NH 3 3 Cl3  4)  Cu  NH 3  4  SO 4
3. In K3  Fe  CN 6  the EAN of Fe 35
17. The no. of moles of AgCl obtained when
excess AgNO 3 is added to one mole of 4. In Cr  NH 3 6  Cl3 the EAN of Cr is 33
[Cr(NH3)4Cl2]Cl
27. According to Sidgwick’s effective atomic
1) 1 2) 2 3) 3 4) 4 number rule the central metal acquires
18. Ligand in a metal carbonyl complex is 1. inert gas configuration
1) NH3 2) CO 3) CN- 4) SCN- 2. octet 3. duplet 4. quartet
19. The no. of moles of AgCl precipitated when
excess of AgNO3 is mixed with one mole of VALENCE BOND THEORY AND SHAPES
[Cr(NH3)3 Cl3] is OF CO-ORDINATION COMPOUNDS
1) 0 2) 1 3) 2 4) 3 28. The shape of the complex species will be
square planar if its coordination number is
NOMENCLATURE 1) 2 2) 6 3) 5 4) 4
20. IUPAC name of the complex CoCl35NH3 is 29. Which of the following is outer orbital complex
1) Cobalt trichloride penta amonium 3
2) Penta amine carbonyl chloride 1) CoF6  2) [Cu ( H 2O )6 ]2
3) Trichloro penta amino cobalt 3) [Co( NH 3 ) 6 ]2 4) Both 1 and 2
4) Pentaaminechlorocobalt (III) chloride
ISOMERISM IN COMPLEXES 30. sp3 d 2 hybridisation is present in
3
21. The property of possessing atleast one atom 1) CoF6  2) [ N i  C O 4 ]
that is attached to four non-identical groups is 2
called 3) [Co( NH 3 ) 6 ]2 4)  Ni  CN 4 
1. polarisation 2. chirality 31. The type of hybridisation present in
3. enantiomerism 4. meridionity 2
22. A racemic mixture has a net rotation Cu  NH 3  4  ion is
1. to right of original plane 1) sp3 2) dsp 2 3) sp 3 d 4) sp3 d 2
2. to left of original plane
3
3. toright or left of original plane 4. zero 32. The shape of CoF6  is
23. Optical isomer have 1) Square Planar 2) Trigonal bipyramidal
1) property of chirality 3) Octahedral 4) Tetrahedral
2) almost identical chemical properties 33. The hybridisation of metal atom & geometry
3) almost identical physical properties 

4) all the above of complex in 


 Ag  NH 3  2

 are
EFFECTIVE ATOMIC NUMBER 1) sp, linear 2) sp2, linear
2
24. The effective atomic number of iron in 3) sp , trigonal planar 4) sp, angular
3 2
 Fe  CN 6  is 34. The magnetic moment of  Ni  Cl 4  is
 
1. 34 2. 36 3. 37 4. 35 1) 2.85BM 2) 1.83BM
25. Which doesnot obey EAN rule 3) 4.86BM 4) 5.95BM
1) Fe  CO 5 2) K 4  Fe  CN 6 
CFT
35. Which of the following system has maximum
3) Cu  NH 3  4  SO4 4) Co  NH 3 6  Cl3 number of the unpaired electrons in an inner
octahedral complex ?
1) d 4 2) d 9 3) d 7 4) d 5
238
SR.INTER - IIT ADVANCED - VOL - 2 238
COMPLEX COMPOUNDS JEE-ADV CHEM-VOL-II
SR-MAIN-CHEM-VOL-II
36. In an octahedral crystal field, the correct set 8. In any complex central metal ion acts as Lewis acid
of low energy orbitals are {electron pair acceptor) and Ligands acts as Lewis
1) d xy , d xz , d x2  y 2 2) d x2  y 2 , d z 2 base {electron pair donar)
9. NH 3 acts as ligand.
3) d xy , d xz , d yz 4) d xy , d x2  y 2
12. In metal carbonyl compounds the oxidation number
37. For the same metal, stabilisation energies of of metal is zero because CO is neutral.
tetrahedral and octahedral complexes are 3
related in presence of strong field ligands is 13. K 3  Fe  CN 6   3K    Fe  CN 6 
1)  t   0 2) t  4   0  6 = 4 moles
3) t  9   0  4 4) t  6   0  4
17. Cr  NH 3 4 Cl2  Cl ionises gives one Cl  ion.
38. The orbitals having lower energy in tetrahedral
complexes according to CFT are So it gives 1 mole AgCl ppt.
1) d xy , d yz , d z 2 2) d xy , d yz , d x2  y 2
20. CO  NH 3 5 Cl  Cl2
3) d xy , d yz , d zx 4) d x2  y 2 , d z 2 21. Chirality property
22. Racemicmixture contains equimolar proportions of
APPLICATION OF COMPLEXES IN d and l forms. So net rotation is zero.
QUALITATIVEANALYSISAND METAL 23. Optical isomers are differ towards rotation of plane
CARBONYLCOMPOUNDSAND STABILITY polarised light.
OF CO-ORDINATION COMPOUNDS 24. EAN=[Z-oxidation number of central ion + 2 x
39. The metal which does not form poly nuclear number of ligands]
carbonyl is = 26 - 3 + 2 x 6 = 35
1) Mg 2) Fe 3) Cr 4) Co 25. EAN = 29 - 2 + 2 x 4 = 35
40. Nessler’s reagent is So, it does not obeys EAN rule.
1) K 2 HgI 4 2) K 2 HgI 2 3) K 2 HgCl4 4) HgI 2 26. EAN = 28 - 2 + 2 x 4 = 34
27. According to Sidgwick’s EAN rule. The central
41. Among the following metal carbonyls, the C-
O bond order is lowest is metal atom acquires inert gas configuration.
28. Shape of complex depends on coordination number.

1) V  CO 6  2)   5 
 Fe CO  If coordination number = 4, shape is tetrahedral
(or) square planar.

3) Cr  CO 6 
  4)  Mn  CO 6  29. NH3 strong field ligand,so it forms inner orbital
complex.
CONCEPTUAL - KEY 30. F- is weak field ligand so it forms outer complex.
32. Coordination number 6. So shape is octahedral.
1) 2 2) 1 3)4 4) 3 5) 2 6) 3 7) 2 34. Cl  is weak field ligand so rearrangement of
8) 1 9) 3 10) 1 11) 3 12) 3 13) 2 14) 2 electron is not possible. Hence Ni 2 ion contains
15) 1 16) 4 17) 1 18) 2 19) 1 20) 4 21) 2 two unpaired electrons.
22) 4 23) 1 24) 4 25) 3 26) 2 27) 1 28) 4 35. In an inner orbital complexes :
29) 4 30) 1 31) 2 32) 3 33) 1 34) 1 35) 1 d 4  t24g eg0 , so two unapired electrons
36)3 37) 3 38) 4 39) 1 40) 1 41) 1
d 9  t2g
6 3
eg , so one unpaired electron
CONCEPTUAL - HINTS
1. Cannot ionise to produce free Cl– (or) SO42
d 7  t26g e1g , so one unpaired electron

2. Co  NH 3 6  Cl3 gives more number ions. d 5  t2g


5 0
eg , so one unpaired electron

7. CO  NH 3 5 Cl  Cl2
239
239 SR.INTER - IIT ADVANCED - VOL - 2
COMPLEX COMPOUNDS JEE-ADV CHEM-VOL-II
SR-MAIN-CHEM-VOL-II

4 7. Which of the following releases metal slowly


37.  t  0 which gives uniform coating in electroplating
9 is
40. Formula of Nessler’s reagent is K 2 HgI 4 . 1) metal salts 2) double salts
41. As increase of electron density back bonding 3) complex salts 4) alums
tendence of  bond increases, so C-O bond order NOMENCLATURE
decreases. 8. IUPAC name for the complex
LEVEL-IA  Cu  NH 3 4  SO 4 is
1) cuprammonium sulphate
WERNERS THEORY 2) copper sulphate tetraammonia
1. Which of the following is not a draw back of 3) tetraamminecopper  II  sulphate
Werner’s theory
4) copper ammonium  IV  sulphate
1) does not explain the valency of metal ions in the
complex ISOMERISM
2) does not give any explanation for the colour of 9. Which of the following does not exhibit optical
complex compounds isomer ?
3) does not explain the magnetic behaviour of 1) Co  NH 3 3 Cl3  2) Co  en 3  Cl3
complex compounds
4) does not correlate electronic configuration of the 3) Co  en 2 Cl2  Cl 4) Co  en  NH 3 2 Cl2  Cl
metal with the formation of complex
10. Co-ordination compounds  Pt  NH 3 3  NCS  
DEFINITION OF and  Pt  NH 3 3  SCN   are example of --
CO-ORDINATION COMPOUND isomerism
2. Which is a coordination compound ? 1) Co-ordination 2) Ionization
1) Ferrous ammonium sulphate 2) Carnallite 3) Linkage 4) Optical
3) Potassium ferrocyanide 4) Gypsum 11. Geometrical isomerism is observed in
1) Tetrahedral complex
3. Aqueous solution of [Co (H2O)5 SO4]Cl gives
2) Square planar complex
precipitate with 3) Linear complex
1) BaCl2 (aq) 2) AgNO3 (aq) 4) Planar triangle complexes
3) both 1 and 2 4) neither 1 nor 2
EFFECTIVE ATOMIC NUMBER
4. Silver Chloride dissolves in excess ammonia 12. Stable complex based on EAN rule
due to the formation of a soluble complex
whose formula is 1) K 4  Fe  CN 6  2) Co  NH 3 5 Cl  Cl2
1) [Ag (NH3)] Cl 2) [Ag (NH3)2] Cl
3) [Ag (NH3)3]Cl 4) [Ag (NH3)4] Cl 3)  Ni  CO 4  4) all the above
5. Zn+2 dissolves in excess of NaOH due to the VBT AND SHAPES OF
formation of COORDINATION COMPOUNDS
1) Soluble Zn (OH)2 2) Soluble Na2 [Zn (OH)4]
13. The hybrdisation of the complex [ NiCl4 ]2 is
3) Soluble Na [Zn (OH)3] 4) ZnO
6. Example of neutral complex compound in the 1) sp3 2) dsp 2 3) sp 3 d 4) sp3 d 2
following is 14. Which of the following has a square planar
1) CoCl3 .6 NH 3 2) CoCl3 .5 NH 3 Geometry?
2 2
3) CoCl3 .4 NH 3 4) CoCl3 .3 NH 3 1) COCl4  2)  PtCl4 
2 2
3)  NiCl4  4)  FeCl4 
240
SR.INTER - IIT ADVANCED - VOL - 2 240
COMPLEX COMPOUNDS JEE-ADV CHEM-VOL-II
SR-MAIN-CHEM-VOL-II
15. Which of the following is paramagnetic ?  
A g  N H 3   N H 3   A g  N H 3  2  ; K 2  1 .7  1 0  3

1) Ni  CO 4 2) Fe  CO 5 
Then, the formation constant of  Ag  NH 3 2 
3) V  C O 6 4) Cr  CO 6
from Ag  and NH 3 is
16. The number of ions formed when
cupraammonium sulphate is dissolved in water 1) 1.7  103 2) 5.92  106
1) 1 2) 2 3) 4 4) zero
3) 1.8  103 4) 1.7  107
CFT
17. Which of the following is correct arrangement
of ligands in terms of field strength
LEVEL-I - A - KEY
1) Cl   F   NCS   NH 3  CN  1) 1 2) 3 3) 2 4) 2 5) 2 6) 4 7) 3
8) 3 9) 1 10) 3 11) 2 12) 4 13) 1 14) 2
2) NH 3  F   Cl   NCS   CN 
15) 3 16) 2 17) 1 18) 4 19) 2 20) 1 21) 2
3) Cl   F   NCS   CN   NH 3 22) 1 23) 2
4) NH 3  CN   NCS   Cl   F 
18. In which of the following octahedral complexes LEVEL-I - A - HINTS
of cobalt (atomic number = 27) will the
3. Co  CN  4 SO4  Cl ionises and gives Cl  ion.
magnitude of  0 be the highest ?
3 3 Ag   Cl   AgCl  ppt 
1) Co  C2O4 3  2) Co  H 2O 6 
3 3 4. AgCl  2 NH 3   Ag  NH 3  2  Cl
3) Co  NH 3 6  4) Co  CN 6 
5. Zn 2  2 NaOH  Na2 ZnO2  H 2
19. If  0  P , the correct electronic configuration
for d 4 system will be ( P =pairing energy) Na2 ZnO2  2 H 2O  Na2  Zn  OH  4 
1) t24g eg0 2) t23g e1g 3) t20g eg4 4) t22g eg2
8. IUPAC name of complex Cu  NH 3  4  SO4 is
APPLICATION OF COMPLEXES IN tetraamminecopper(II) sulphate.
QUALITATIVEANALYSISAND METAL
CARBONYLCOMPOUNDSAND STABILITY 12. EAN of  Ni  CO  4  = 28 - 0 + 2 x 4 = 36, So it
OF CO-ORDINATION COMPOUNDS is stable
20. Ammonium ions are detected with 13. It is outer orbital complex. Coordination number
1) Nessler’s reagent 2) Borsch reagent is 4. Shape is tetrahedral.
3) Tollen’s reagent 4) Fehling’s solution
18. Strength of ligands
21.  Ph3 P 3 RhCl  is a familiar catalyst used in
C2O42  H 2O  NH 3  CN 
1) hydrogenation of oils
2) hydrogenation of alkenes 19. Weak field ligand
3) dehydration of alcohols
23. K  K1.K 2 .
4) dehydration of aldehydes
22. Metals those can be extracted with aqueous
solution of sodium cyanide as complexes are
1) Au and Ag 2) Fe and Ag
3) Au and Hg 4) Hg and Fe

23. Ag   NH 3  Ag  NH 3  ; K1  3.5  103

241
241 SR.INTER - IIT ADVANCED - VOL - 2
COMPLEX COMPOUNDS JEE-ADV CHEM-VOL-II
SR-MAIN-CHEM-VOL-II
9. IUPAC name of Li[AlH4] is
LEVEL-IB
1) Lithium aluminium hydride
WERNERS THEORY
2) Lithium tetrahydrido aluminate [III]
1. The complex formed by the combination of
calcium ions and ethylene di ammine tetra 3) Tetrahydride aluminium lithionate
4 4) Aluminium lithium hydride
acetate.  EDTA  Number of moles of calcium
ISOMERISM
ions produced by dissolving of one mole of that
10. Dipole moment will be zero in the complexes
complex in excess of water is
2
1) one 2) two 3) four 4) five 1) Ni  CN  4  2) Cis  Pt NH 3  2 Cl 2 
DEFINITION OF
3) Trans  Pt NH 3  2 Cl 2  4) Both 1 and 3
CO-ORDINATION COMPOUND
2. Which is a double salt? 11. The number of geometrical isomers of
1) Carnalite 2) Potassium ferrocyanide Co  NH 3 3  NO3 3  is
3) Potasium ferricyanide 4) Nessler’s reagent
1) 0 2) 2
4) 4 3) 3
3. Bonds present in K4 [Fe (CN)6] are

1) Only ionic 2) Only covalent 12. For the given comlex CoCl2  en  NH 3 2  ,
3) Ionic and Covalent
the number of geometrical iosmers, the number
4) Ionic, covalent and coordinate covalent
of optical isomers and total number of isomers
4. Copper sulphate solution forms blue coloured of all type possible respectively are
complex with excess of ammonia. Its formula
is 1) 2, 2 and 4 2) 2, 2 and 3
1) [Cu (NH3)4]+3 2) [Cu (NH3)4]+2 3) 2, 0 and 2 4) 0, 2 and 2
3) [Cu (NH4)3]+2 4) [Cu (NH4)3]+3 EFFECTIVE ATOMIC NUMBER
5. Which of the following cannot act as a ligand 13. Which does not obey EAN rule?
1) BF3 2) NH3
3) NO  4) CN 
1) K 4  Fe  CN 6  2) K 3  Fe  CN 6 
6. Alum in aqueous solution gives positive test
for
3) Co  NH 3 6  Cl3 4)  Ni  CO 4 
2
A) K  B) Al 3 C) SO 4
VBT AND SHAPES OF
1) A only 2) B only 3) A and B 4) A, B and C
COORDINATION COMPOUNDS
7. Chelates are used in
14. The following solutions requires three moles
1) Analytical chemistry 2) Water softning of AgNO3 for the complete precipitation of all
3) Removal of Pb2 from the blood the chloride ions present in it
1) One litre of 1M [Co (NH3)6] Cl3
4) All of these
2) Three litres of 1M [Co (NH3)4 Cl2] Cl
NOMENCLATURE 3) One litre of 1.5M [Co (NH3)5 Cl] Cl2
4) All the above
8. Hexaaquotitanium  III  Chloride is 15. AgCl dissolved in excess of NH , KCN and
3
represented as Na 2S 2O 3 solutions the complex produces
1)  Ti  H 2 O 6  Cl3 2)  Ti Cl 3  6H 2 O ions.
  3
1.  Ag  NH3 2  ,  Ag  CN 2  &  Ag  S2O3 2 
3)  Ti  H 2 O 5 Cl  Cl 2 4)  Ti  H 2 O 3 Cl3 
2 3 2
2.  Ag  NH 2 2  ,  Ag  CN  2  &  Ag 4  S 2 O3 5 

242
SR.INTER - IIT ADVANCED - VOL - 2 242
COMPLEX COMPOUNDS JEE-ADV CHEM-VOL-II
SR-MAIN-CHEM-VOL-II
2 
3.  Ag  NH 3 2  ,  Ag  CN 2  &  Ag 2  S 2 O3 2 
2 LEVEL-I B - HINTS
 3 2 1. formula of the compound is  Ca2  EDTA  
4.  Ag  N H 3  4  ,  Ag  C N 4  &  Ag 2  S 2 O 3 2 
16. The secondary valency of chromium in 4. CuSO4  4 NH 3  Cu  NH 3  4  SO4
Cr  en 3  Cl3 is 5. BF3 Lewis acid. It can’t donate the lone pair of
1) 6 2) 3 3) 2 4) 4 electrons.
8. Hexaaquotitanium(III) chloride is given as
CFT
17. Stabilisation energy of octahedral complex with Ti  H 2O 6  Cl3 .
7 configuration
d 12. Two geometrical isomers, C is & trans, for C is
A) 1.8  0 with one unpaired electron isomer two optical isomers are possible. Therefore
total isomers are 3.
B) 1.8  0 with three unpaired electrons 13. EAN = Z  x  n  y ; Z=At number of metal ion,
C) 0.8  0 with one unpaired electron x=charge of metal ion, n= secondary valency
14. n = M x V
D) 0.8  0 with three unpaired electrons
1) A and D 2) A and B 3) C and D 4) B and C 15. AgCl  2 NH 3   Ag  NH 3  2  Cl
18. If  0  P , the correct electronic confuguration AgCl  2 KCN  K  Ag  CN 2   KCl
for d system will be ( p = pairining energy)
4
AgCl  2 Na2 S 2O3  Na3  Ag  S 2O3 2   KCl
1) t24g eg0 2) t23g e1g 0 4
3) t2g eg 4) t22g eg2
17. For strong field ligands -
APPLICATION OF COMPLEXES IN CFSE   6  0.4  1 0.6   1.8 0
QUALITATIVEANALYSISAND METAL
one unpaired electron
CARBONYLCOMPOUNDSAND STABILITY For weak field ligands
OF CO-ORDINATION COMPOUNDS
19. The ligand used in the identification of cupric CFSE   5  0.4  2  0.6   0.8 0
copper in the laboratory three unpaired electrons
1) NH 3 2) I  3) CN  4) S2O32 18. Strong field ligand.

20. The cofiguration of an element ‘X’ is 4 s1 3d 10 . ADDITIONAL QUESTIONS


The wrong statement regarding the element 1. Type of Pair of examples
‘X’ is isomerism
1) it forms complexes
2) it exhibits variable velency A)Ionisation (a) Cr  H 2O 6  Cl3 and
3) it forms paramagnetic ions only
4) It can form coloured salts Cr  H 2O 5 Cl  Cl2 .H 2O
 
21. In the qualitative analysis of group 3 cations
blood red colouration is a test for Cr  NH 3 6  Co  CN 6 
1) iron using cyanide as ligand B) Linkage (b)   

2) chromium using cyanide as ligand & Co  NH 3 6  Cr  CN 6 


3) iron using thiocyanide as ligand
4) chromium using thiocyanide as ligand Co  NH 3 5  NO2   Cl2
 
C)Coordination (c)  & Co  NH 3 5  ONO   Cl2

LEVEL-I B - KEY
1) 2 2) 1 3) 4 4) 2 5) 1 6) 4 7) 4 Co  SO4  NH 3 5  Br
 
8) 1 9) 2 10) 4 11) 2 12) 2 13) 2 14) 4 D)Hydrate (d)
& Co  Br  NH 3 5  SO4
15) 1 16) 1 17) 1 18) 1 19) 4 20) 3 21) 3

243
243 SR.INTER - IIT ADVANCED - VOL - 2
COMPLEX COMPOUNDS JEE-ADV CHEM-VOL-II
SR-MAIN-CHEM-VOL-II
The correct match is 5. Match the following.
A B C D A B C D Set -I Set -II
1. a b c d 2. b a d c A) Co-ordination number 3 1) Pentagonal
3. d c b a 4. d b c a bipyramidal
2. COLUMN-I COLUMN-II B) Co-ordination number 2 2) Trigonal
(Complex) (Geometry) bipyramidal
A)  Ni (CN )6 2  p) Tetrahedral C) Co-ordination number 5 3) Linear
D) Co-ordination number 7 4) Trigonal planar
B)  ZnCl4 2  q) Tetragonal
The correct matching is
3
C) Co(en)3  r) Sqaure planar A B C D A B C D
D) Cu ( NO2 ) 6 4  s) square pyramid 1) 4 3 2 1 2) 1 2 3 4
3) 4 3 1 2 4) 3 1 2 4
t) Octahedral
A B C D A B C D 6. COLUMN-I COLUMN-II

1) r p t p 2) r p t q,t (Complex Ions) (No. of unpaired


electrons)
3) p q r s 4) p t q s
3. COLUMN-I COLUMN-II A) CrF6 4  p)One
(Equivalent (formula) B)  MnF6 4  q) Two
conductance)
C) Cr (CN )6 4  r) Three
A) 229 p)  Pt ( NH 3 )5 Cl  Cl3

B) 97 q)  Pt ( NH 3 )3 Cl3  Cl D)  Mn(CN )6 4  s) Four


t) Five
C) 404 r)  Pt ( NH 3 )4 Cl2  Cl2
A B C D A B C D
D) 523 s)  Pt ( NH 3 )6  Cl4
1) s t q p 2) q r s t
A B C D A B C D 3) t q s p 4) p q r s
1) s p q r 2) r q s p
7. COLUMN-I COLUMN-II
3) r q p s 4) s p r q
(Complex) (O.N. of Co)
4. COLUMN-I COLUMN-II
(Complex) (  effective) A) Co( NCS )( NH 3 )5  (SO3 ) p) -1

A) K3  MnF6  p) 1.70 B) Na Co(CO)4  q) 0

B) K3 VF6  q) 2.8 C) Na4 Co(S2 O3 )3  r) 1

C) K2  Mn(CN )6  r) 3.8 D) Co2 (CO)8 s) 2

D) K3 TiF6  s) 4.9 t) 3
A B C D A B C D A B C D A B C D
1) s q r p 2) q s r p 1) t p s q 2) q t s p
3) s q p r 4) p q r s 3) p t s q 4) q r s t

244
SR.INTER - IIT ADVANCED - VOL - 2 244
COMPLEX COMPOUNDS JEE-ADV CHEM-VOL-II
SR-MAIN-CHEM-VOL-II
8. Match the lists I and II and pick the correct
matching from the codes given below LEVEL-IIA
List - I List - II
(complex) (Structure and magneticmoment) DEFINITION OF
a)  Ag  CN 2 

1) square planar and 1.73 BM
CO-ORDINATION COMPOUND
3 1. Oxidation number of cobalt in K Co  CO 4 
b) Cu  CN  4  2) Linear and zero
is
4
c) Cu  CN 6  3) Octahedral and zero 1) +1 2) +3 3) -1 4) -3
2
d) Cu  NH 3  4  4) tetrahedral and zero
2.  EDTA4 is a
1) Monodentate ligand 2) Bidentate ligand
4
e)  Fe  CN 6  5) octrahedral and 1.73 BM 3) Quadridentate ligand 4) Hexadentate ligand
1) a-2,b-4,c-5,d-1,e-3 2) a-5,b-4,c-1,d-3,e-2 3. ‘en’ is an example of a
3) a-1,b-3,c-4,d-2,e-5 4) a-4,b-5,c-2,d-1,e-3 1) Monodentate ligand 2) Bidentate ligand
ASSERTION & REASON TYPE 3) Tridentate ligand 4) Hexadentate ligand
Two statements ‘A’ and ‘R’ are given below: Select NOMENCLATURE
your answers to these items using the codes given
below : 4. IUPAC name of K 2  PtCl6  is
1) Both A and R are correct and R is the correct
1) Potassiumhexachloroplatinum
explanation of A
2) Both A and R are correct and R is not a correct 2) Potassiumhexachloroplatinum(IV)
explanation of A 3) Potassium hexachloroplatinate(IV)
3) A is true but R is false4) A is false but R is true 4) Dipotassium hexa chloro platinum
9. Assertion (A) : Among the cobalt (III) complexes 5. IUPAC name of
3 3
Co  NH 3 6  is a diamagnetic but CoF6  is  Pt  NH 3 3 Br  NO2  Cl  Cl is
paramagnetic.
1) Triamminechlorobromonitroplatinum (IV)
3
Reason (R) : Hybridisation of Co  NH 3 6  is chloride
3 2) Triamminebromonitrochloroplatinum (IV)
sp 3 d 2 where as hybridisation of Co in CoF6  chloride
is d 2 sp 3 . 3) Triamminebromochloronitroplatinum (IV)
10. Assertion (|A) : The complex ion chloride
 4) Triamminenitrochlorobromoplatinum (IV)
cis  Co  en 2 Cl2  is optically active
chloride
Reason (R) : It is an octahedral complex 6. Tetrammine diaqua copper (II) hydrox
11. Assertion (A) :  Ni  CO 4  is diamagnetic and ide is given by the formula
tetrahedral in shape 1) Cu  NH 3  4   OH 2 .2 H 2O
Reason (R) : Hybridisation of the complex is
dsp 2 . 2)  Cu  NH 3 4  OH 2  .2 H 2 O

ADDITIONAL QUESTIONS - KEY 3) Cu  NH 3  4  H 2 O  2   OH  2


1) 3 2) 1 3) 3 4) 1 5) 1 6) 1 7) 1
8) 1 9) 3 10) 2 11) 3 4)  Cu  NH 3 4  H 2 O  OH 2 

245
245 SR.INTER - IIT ADVANCED - VOL - 2
COMPLEX COMPOUNDS JEE-ADV CHEM-VOL-II
SR-MAIN-CHEM-VOL-II
ISOMERISM VBT
7. The complexes Co  NH 3 6  Cr  C2O4 3  12.  FeF6 3 has Fe atom .... hybridized with
unpaired ........ electrons.
and Cr  NH 3 6  Co  C2O4 3 
1) d 2 sp 3 , 4 2) d 2 sp 3 , 5
1) Geometrical isomerism 2) Ionization energy
3) sp 3 d 2 , 5 4) sp 3 d 2 , 3
3) Co-ordination isomerism 4) Linkage isomerism
13. How many EDTA molecules are required to
8. Which of the following complex or the complex
ion will show geometrical isomerism? make an octahedral complex with a Ca 2 ion?

1) two 2) six 3) three 4) one
1)  Pt  NH 3  2 Cl2  2)  Pt  NH 3  Cl5  14. Which of the complexes will exhibit the
minimum paramagnetic behaviour
3
3)  Pt  NH 3 5 Cl  4) Co  NH 3 6  Cl3 2 2
1)  Fe  H 2O 6  2)  Mn  H 2O 6 
9. Which isomerism is exhibited by 2 2
3) Cr  H 2O 6  4)  Ni  H 2O 6 
Co  NH 3 3  H 2O 3  Cl3
15. The expected spin only magnetic moments for
1) Geometrical isomerism 2) Linkage isomerism 4 3
3) Coordination isomerism4) Ionization isomerism 
 Fe  CN  
6
and  FeF6  are
1) 1.73and1.73 B.M 2) 1.73and 5.93 B.M
10. Co  NH 3 4  NO2 2  Cl exhibits
3) 0.0 and1.73 B.M 4) 0.0 and 5.92 B.M
1) Linkage isomerism, ionization isomerism and
geometrical isomerism 16. The volume (in mL) of 0.1M AgNO3 required
2) Ionisation isomerism, geometrical isomerism and for complete precipitation of chloride ions
optical isomerism present in 30 mL of 0.01M solution of
3) Linkage isomerism, geometrical isomerism and Cr  H 2O 5 Cl  Cl2 as silver chloride is close
optical isomerism
to
4) Linkage isomerism, ionization isomerism and 1) 3 2) 4 3) 5 4) 6
optical isomerism. 17. Among the following ions which one has the
11. Which one of the following is an example of highest paramagnetism:
coordination isomerism? 3 2
1) Cr  H 2O 6  2)  Fe  H 2O 6 
1) Co  NH 3 5 Br  SO4 and
2 2
3) Cu  H 2O 6  4)  Zn  H 2O 2 
Co  NH 3 5 SO4  Br
18. Which of the following complexes is an outer
2) Co  NH 3 5 NO2  Cl2 and orbital complex?
4 4
1)  Fe  CN 6  2)  Mn  CN 6 
 Co  NH 3 5 ONO  Cl2
3 2
3) Co  NH 3 6  4)  Ni  NH 3 6 
3) Cr  H 2O 6  Cl3 and
EFFECTIVE ATOMIC NUMBER
Cr  H 2O 5 Cl  Cl2 .H 2O 19. The EAN of cobalt in the complex ion

4) Cr  NH 3 6  Co  CN 6  and Co  en 2 Cl2  is
1) 27 2) 36 3) 33 4) 35
Co  NH 3 6  Cr  CN 6 

246
SR.INTER - IIT ADVANCED - VOL - 2 246
COMPLEX COMPOUNDS JEE-ADV CHEM-VOL-II
SR-MAIN-CHEM-VOL-II
CFT 14. H 2O is weak field ligand so, rearrangement electron
20. For an octahedral complex, which of the is possible
following d electron configuration will give
maximum crystal - field stabilisation energy 1) n  4    4.85 BM
1) High spin d 6 2) Low - spin d 4 2) n  5    5.92 BM
3) Low spin d 5 4) High - spin d 7
3) n  4    4.85 BM
APPLICATION OF COMPLEXES IN
QUALITATIVEANALYSISAND METAL 4) n  2    2.8 BM
CARBONYLCOMPOUNDSAND STABILITY 16. 1 - 2
OF CO-ORDINATION COMPOUNDS 0.3 - ?
21. Chromium compound widely used in tanning
of leather is 0.3 0.6
 2  0.6 ; 0.6  0.1 X ; X   6ml
1) Cr2O3 2) Cr2O2Cl2 3) Cr2O3 1 0.1

4) K2 SO4Cr2  SO4 3 24H 2O 17. 1) n  3 2) n  4 3) n  1 4) n  0


18. Ni 2  4 s 0 3d 8 rearrangement of electron is not
22. Wilkinsons catalyst,  Rh  Ph3 P 3 Cl  is used
for possible. So, hybridisation is sp3 d 2 .
1) Hydrogenation of oils
20. High spin d 6 : t24g eg2
2) Hydrogenation of alkynes
3) Hydrogenation of alkenes
CFSE   0.4  4  0.6  2  0
4) Polymerisation of alkenes
23. Coordination compounds have great
importance in biological systems. In this Low spin d 4 : t24g eg0
context which of the following statements is
incorrect? CFSE =  0.4  4  1.6 0
1) Chlorophylls are green pigments in plants and
contain calcium. Low spin d 5 : t25g eg0
2) Haemoglobin is the red pigment of blood and
contains iron. CFSE =  0.4  5  0
3) Cyanocobalamin is B12 and contains cobalt. High spin d 7 : t25g eg2
4) Carboxypeptidase-A is an enzyme and contains
zinc. CFSE =  0.4  5  0.6  2  0
LEVEL-II - A - KEY 22. Wilkinson’s catalyst is  Ph3P 3 RhCl
1) 3 2) 4 3) 2 4) 3 5) 3 6) 3 7) 3
8) 1 9) 1 10) 1 11) 4 12) 3 13) 4 14) 4
15) 4 16) 4 17) 2 18) 4 19) 2 20) 3 21) 4
22) 3 23) 1

LEVEL-II - A - HINTS
2.  CH COO 
3

2
 N  CH 2  CH 2  N 

 CH COO 
3

2

12. F  is weak field ligand so, its forms outer complex.

247
247 SR.INTER - IIT ADVANCED - VOL - 2
COMPLEX COMPOUNDS JEE-ADV CHEM-VOL-II
SR-MAIN-CHEM-VOL-II

LEVEL-IIB 7. Which of the following is not optically active?


3 3
DEFINITION OF 1) Co  en 3  2) Cr  ox 3 
CO-ORDINATION COMPOUND

1. An ambidentate ligand is one which 3) cis  CoCl2  en 2 
1) is linked to the metal atom at two points

2) has two donor atoms but only one of them has 4) trans  CoCl2  en  2 
the capacity to form a coordinate bond.
3) has two donor atoms but either of the two can 8. Cis-trans isomerism is found in square planar
form a co-ordinate bond complexes of the molecular formula: (a and b
4) forms chelate rings are monodentate ligands)
2. Which of the following is not chelating agent? 1) Ma4 2) Ma3b 3) Ma2b2 4) Mab3
1) Thiosulphato 2) Oxalato 9. Which of the following isomeric pairs shows
3) Glycinato 4) Ethylene diamine ionization isomerism?

NOMENCLATURE 1) Co  NH 3 6  Cr  CN 6  and

3. IUPAC name of Na3 CoCl  NO2 5  is Cr  NH 3 6  Co  CN 6 

1) Sodium chloropentanitrocobaltate(III) 2) Cr  H 2O 6  Cl3 and


2) Sodium cobaltnitrate
Cr  H 2O 5 Cl  Cl2 .H 2O
3) Trisodium chloropentanitro cobalt
4) Pentanitrocobalt (III) trisodium complex 3)  Pt  NH 3 2 Cl 2  and
4. The IUPAC name of the Wilkinsons catalyst
 Pt  NH 3 4   PtCl4 
 RhCl  P Ph3 3  is
4) Co  NH 3 5 Br  SO4 and
1) Chlorotris (triphenylphopshine) rhodium (I)
2) Chlorotris (triphenylphosphine) rhodium (IV) Co  NH 3 5 SO4  Br
3) Chlorotris (triphenylphosphine) rhodium (0) VBT
4) Chlotrotris (triphenylphosphine) rhodium (VI) 10. List - I List - II
5. IUPAC name for the compound A) Ti 3 1) Charge transfer phenomenon\
Co  NH 3 5  NO2   Cl2 is B) MnO4 2) Impurities
1)Nitrito-N-pentaammine cobalt (III)chloride C) F2 3) s-s transition
2)Nitrito-N-pentaammine cobalt (II) chloride D) Gems 4) d-d transition
3) Pentaammine nitrito-N-cobalt (II) chloride 5) Excitation of electron \
4) Pentaammine nitrito-N-cobalt (III)chloride The correct match which is responsible for
colour
ISOMERISM 1) A - 4, B - 1, C - 2, D - 5
6. Both geometrical and optical isomerism are 2) A - 4, B - 1, C - 5, D - 2
shown by 3) A - 4, B - 5, C - 1, D - 2
 2
4) A - 5, B - 2, C - 3, D - 4
1) Co  en 2 Cl2  2) Co  NH 3 5 Cl  11. Co-ordination number of Cr is 6.A complex
 3
entity with C2O42 en, superoxide as ligands is
3) Co  NH 3  4 Cl2  4) Cr  ox 3 
Cr  C2O4   en   O2   . The ratio of x:y:z is
(transform)  x y z

1) 1 : 1 : 2 2) 1 : 1 : 1 3) 1 : 2 : 2 4) 2 : 1 : 1

248
SR.INTER - IIT ADVANCED - VOL - 2 248
COMPLEX COMPOUNDS JEE-ADV CHEM-VOL-II
SR-MAIN-CHEM-VOL-II

12. The geometries of Ni  CO 4 and 20. Ziegler - Natta catalyst is


1) Solution of SnCl4 + trialkylaluminium
Ni  PPh3  2 Cl2 are
2) Solution of TiCl4 + trialkylaluminium
1) both square planar
2) tetrahedral and square planar 3) Solution of TiCl4 + trialkylchromium
3) both tetrahedral
4) Solution of SnCl4 + Tollen’s reagent
4) square planar and tetrahedral
13. What are the magnetic moment (in BM) for LEVEL-IIB - KEY
Ni(II) ion in square planar and octahedral 1) 3 2) 1 3) 1 4) 1 5) 4 6) 1 7) 4
geometry, respectively?
8) 3 9) 4 10) 2 11) 1 12) 2 13) 1 14) 3
1) 0 and 2.83 2) 2.83 and 2.83
3) 0 and 1.73 4) 2.83 and 0 15) 1 16) 1 17) 3 18) 4 19) 3 20) 2
14. A solution containing 2.675g of CoCl3 . 6NH 3 LEVEL-IIB - HINTS
(molar mass = 267.5 g mol 1 ) is passed through 11. C O 2 and en are bidentate ligands O  is
2 4 2
a cation exchanger. the chloride ions obtained monotente ligand.
is solution were treated with excess of AgNO3 12. Hybridisations are 3 and
sp dsp 2
to give 4.78 g of AgCl (molar mass = 143.5 g 13. For square planar
mol 1 ). The formula of the complex is
(At. mass of Ag = 108 u) n0
1) CoCl3  NH 3 3  2) CoCl  NH 3 5  Cl2 For octahedral

3) Co  NH 3 6  Cl3 4) CoCl2  NH 3 4  Cl n2


2
15. The complex  Pt  NH 3  4  has... structure 2.675
14. Moles of the complex =  0.01
1) square planar 2) tetrahedral 267.5
3) pyramidal 4) pentagonal 4.78
moles of AgCl precipitated =  0.033
CFT 143.5
16. Crystal field stabilization energy for high spin Thus, 1 mole of the complex will precipitate
d 4 octahedral complex is 0.033
AgCl  3moles . That means 1 molecule of
1) 0.6 0 2) 1.8 0 3) 1.6 0 4) 1.2 0 0.01
APPLICATION OF COMPLEXES IN the complex contains 3 ionizable Cl.
QUALITATIVEANALYSISAND METAL Hence, the formula is Co  NH 3 6  Cl3
CARBONYLCOMPOUNDSAND STABILITY
OF CO-ORDINATION COMPOUNDS 16. Weak field ligand d 4  t 23g , e1g
17. Which of the following is antidote for lead CFSE = 3  0.4  1 0.6  0.6 0
poisoning?
1) CoCl3 2) Cis-platin 3) EDTA 4) DMG PREVIOUS EAMCET QUESTIONS
18. EDTA is used for the estimation of 1. CO  NH 3 5 SO4  Br and CO  NH 3 5 Br  SO4
1) Na  and K  ions 2) Cl  and Br  ions are a pair of ____ isomers [EAM-2008]

3) Cu and Ag ions 4) Ca and Mg ions
2 2 2 1) Ionization 2) Ligand
19. Wilkinson’s catalyst used as a homogeneous 3) Coordination 4) Hydrate
catalyst in the hydrogenation of alkenes
contains
1) iron 2) aluminium 3) rhodium 4) cobalt
249
249 SR.INTER - IIT ADVANCED - VOL - 2
COMPLEX COMPOUNDS JEE-ADV CHEM-VOL-II
SR-MAIN-CHEM-VOL-II
2. The solution of X having excess of caustic
LEVEL-III
potash is used to detect ammonia. Which of
the following is X [EAM-2011] WERNER’S THEORY
1. One mole of the complex compound
1) K 2  HgI 4  2) CO  NH 3 6  Cl3 Co  NH 3 5 Cl3 gives 3 moles of ions on
3) K 3  Fe  CNS 6  4) CO  NH 3 5 SO4  Br dissolution in water. One mole of the same
complex reacts with two moles of AgNO3
3. A coordinate complex contains CO 3 , Cl  and
solution to yield two moles of AgCl  s  . The
NH 3 when dissolved in water, one mole of this structure of complex is
complex gave a total of 3 moles of ions. The
1) Co  NH 3 4 Cl  Cl2 .NH 3
complex is [EAM-2012]
2) Co  NH 3 5 Cl  Cl2
1) Co  NH 3 6  Cl3 2) Co  NH 3 5 Cl  Cl2
3) Co  NH 3 3 Cl3  2.NH 3
3) Co  NH 3 4 Cl2  Cl 4) Co  NH 3 3 Cl3 
4) Co  NH 3 4 Cl2  Cl.NH 3
4. Identify the correct set [EAM-2013]
Molecule Hybridisation Shape 2. The primary and secondary valencies of
chromium in the complex ion,
1) PCl5 dsp 3 square pyramidal dichlorodioxalatochromium (III), are
2 respectively
2)  Ni  CN 4  sp 3 tetrahedral 1) 3,4 2) 4,3 3) 3,6 4) 6,3
3) SF6 sp 3 d 2 octahedral 3. In the complex with formula MCl3 .4 H 2O the
4) IF3 dsp 3 pyramidal co-ordination number of the metal M is six and
there is no molecule of hydration in it. The
5. Match the following [EAM-2014]
List-I List-II volume of 0.1M AgNO3 solution needed to
3 precipitate the free chloride ions in 200ml of
A) sp 3 I) Co  NH 3 6  0.01M soltuion of the complex is
1) 40ml 2) 20 ml 3) 60 ml 4) 80 ml
B) dsp 2 II)  Ni  CO  4  4. The molar ionic conductances of octahedral
complexes.
C) sp 3 d 2 III)  Pt  NH 3  2 Cl2 
I) PtCl4 .5 NH 3 II) PtCl4 .4 NH 3
3
D) d 2 sp 3 IV)  CoF6  III) PtCl4 .3NH 3 IV) PtCl4 .2 NH 3
V)  Fe  CO 5  1) I<II<III<IV 2) IV<III<II<I
3) III<IV<II<I 4)IV<III<I<II
1) A-II, B-III, C-IV, D-I
2) A-II, B-III, C-I, D-V DEFINITION OF
3) A-III, B-II, C-IV, D-I CO-ORDINATION COMPOUND
4) A-V, B-II, C-IV, D-III 5. The coordination number of a central metal
atom in a complex is determined by
6. Ti  H 2O 6  Cl3 has Ti atom ___ hybridized 1) the number of ligands around a metal ion bonded
with unpaired ___ electrons. [EAM-2010] by sigma bonds
2) the number of ligands around a metal ion bonded
1) d 2 sp 3 ,1 2) d 2 sp 3 , 4
by  -bonds
3) sp 3 d 2 ,1 4) sp 3 d 2 , 3 3) the number of ligands around a metal ion bounded
by sigma and pi bonds both.
PREVIOUS EAMCET - KEY 4) the number of only anionic ligands bonded to
1) 1 2) 1 3) 2 4) 3 5) 1 6) 3 the metal ion.

250
SR.INTER - IIT ADVANCED - VOL - 2 250
COMPLEX COMPOUNDS JEE-ADV CHEM-VOL-II
SR-MAIN-CHEM-VOL-II
6. Among the following which are ambidentate 12. Of the following configurations, the optical
lignads ? isomers are
a) NO b) NO

c) EDTA 4-
2 3
en en
d) C O
2 e) SCN
4
2
f) H NCH CH NH

2 2 2 2 Cl Cl
1) (a) and (b) 2) (c) and (d)
3) (a) and (f) 4) (a) and (e)
7. The number of donor sites in dimethyl
glyoxime, glycinato, diethylene triamine and Cl Cl Cl
EDTA are respectively en en
1)2,2,3 and 6 2) 2,2,3 and 4
3) 2,2,2 and 6 4) 2,3,3 and 6 (I ) (II )
NOMENCLATURE en
8. The IUPAC name of the coordination en
Cl Cl
compound K 3  Fe  CN 6  is
1) potassium hexacyanoferrate (II)
2) potassium hexacyanoferrate (III) Cl Cl
3) potassium hexacyanoiron (II) en en
4) tripotassium hexacyanoiron(II)
ISOMERISM (III ) (IV )
9. Which one of the following has largest number 1) I and II 2) I and III 3) II and IV 4) II and III
of isomers? 13. The type of isomerism shown by
 2
1)  Ru  NH 3  4 Cl2  2) Co  NH 3 5 Cl  Co  NH3 4 Br2  Cl is

2  1) Optical and ionisaton


3)  Ir  Ph3 2 H  CO   4) Co  en 2 Cl2  2) Geometrical and optical
(R=alkyl group, en=ethylenediamine) 3) Geometrical and ionisation
10. Which of the following compounds shows 4) Only geometrical
optical isomerism? 14. Which of the following doesn’t have optical
3 3 isomer?
1) Cr  C2O4 3  2) Co  CN 6 
1) Co  en 3  Cl3 2) Co  NH 3 3 Cl3 
2 2
3) Cu  NH 3  4  4)  ZnCl4 
3) Co  en 2 Cl2  Cl 4) Co  en  NH 3 2 Cl2  Cl
11. In which of the following pairs both the
complexes do not show optical isomerism? VBT AND SHAPES OF
3 COORDINATION COMPOUNDS
1) cis  Cr  C2O4 2 Cl2  ,
15. The complex K 3  Fe  CN 6  should have a
trans  Co  NH 3  4 Cl2 
spin only magnetic moment of
2) Co  en 3  Cl3 , Cis  Co  en 2 Cl2  Cl 1) 48BM 2) 2 5BM 3) 35BM 4) 6BM
2 16. Which one of the following complexes is an
3)  PtCl2  en   ,  NiCl2 Br2 
outer orbital complex?
4) Co  NO3 3  NH 3 3  , cis   Pt  en  2 Cl2  1)  Fe  CN 6 
4
2)  Mn  CN 6 
4

3 2
3) Co  NH 3 6  4)  Ni  NH 3 6 

251
251 SR.INTER - IIT ADVANCED - VOL - 2
COMPLEX COMPOUNDS JEE-ADV CHEM-VOL-II
SR-MAIN-CHEM-VOL-II
[Atomic numbers: Mn = 25, Fe = 26, Co = 27, Ni 4) Square planar, tetrahedral ,octahedral :
= 28] dsp 2 , sp3 , sp3d 2 :0,5.9,4.9
17. The correct order of magnetic moments (spin
only values in bohr magneton) among is
CFT
2 2 4 22. Cr  H 2O 6  Cl3 has a magnetic moment of
1)  MnCl4   CoCl4    Fe  CN 6 
3.83BM. The correct distribution of 3d
2 4 2 electrons in chromium present in the complex
2)  MnCl4    Fe  CN 6   CoCl4 
is
4
3)  Fe  CN 6    MnCl4   CoCl4 
2 2
1) 3 d 1xy , 3 d 1yz , 3d zx
1
2) 3 d 1xy , 3 d 1yz , 3d 1z 2
1 1 1 1 1 1
4 2
4)  Fe  CN 6   CoCl4    MnCl4 
2 3) 3d x2  y 2  , 3d z 2 , 3d xz 4) 3d xy ,3d  x2  y 2  ,3d yz

(Atomic numbers: Mn = 25, Fe = 26, Co = 27) 23. In which of the following octahedral complexes
18. Which one of the following cyano complexes will the magnitude of  0 be the highest
would exhibit the lowest value of paramagnetic 3 3
behaviour? 1) Co  CN 6  2) Co  C2O4 3 
3 3 3 3
1)  Fe  CN 6  2) Co  CN 6 
3) Co  H 2O 6  4) Co  NH 3 6 
3 3 24. Which of the following is a correct Irving-
3) Cr  CN 6  4)  Mn  CN 6 
Williams order? (Tendency of complex
19. Which of the following statemants is not formation)
correct? 1) Mn 2   Fe 2   Co 2   Ni 2 
2
1) The complexes  NiCl  and  Ni  CN   differ in
4
2
2) Ni 2   Co 2   Fe 2   Mn 2 
4

their magnetic properties. 3) Fe 2   Mn 2   Ni 2   Co 2 


2 2
2) The complexes NiCl  and  Ni  CN   differ in the
4 4) Co 2   Mn 2   Fe 2   Ni 2 
4

state of hybridisation of nickel. 25. Which order is correct in spectrochemical


2
3) the complexes Ni Cl  and  Ni  CN   differ in
4
2
series of ligands?4

geometry. 1) Cl   F   C O 2  NO  CN  2 4 2
2 2
4) The complexes Ni Cl  and  Ni  CN   differ in
4 4
2) CN   C2O42  Cl   NO2  F 
primary valancies of nickel.
20. The species having tetrahedral shape is 3) C2O42  F   Cl   NO2  CN 
2 2
1)  PdCl  4
2
2)  Ni  CN 4   3)  Pd  CN 4   4)  NiCl  4
2

2
4) F   Cl   NO2  CN   C2O42
21. and  FeF  . Geometry,
 Ni  CN  4  ,  MnBr4 
2
y, 6
3

hybridisation and magnetic moment of the ions APPLICATION OF COMPLEXES IN


respectively are QUALITATIVEANALYSISAND METAL
1) Tetrahedral ,square planar, octahedral : CARBONYLCOMPOUNDSAND STABILITY
sp3 , dsp 2 , sp3d 2 :5.9,0,4.9 OF CO-ORDINATION COMPOUNDS
2) Tetrahedral ,square planar, octahedral : 26. Which of the following ligands is called  -
acceptors?
dsp 2 , sp3 , sp3d 2 :0,5.9,4.9
3) Square planar, tetrahedral ,octahedral : CO, CN  , NO 
dsp 2 , sp3 , d 2 sp3 :5.9, 4.9,0 I   II   III 
1) I,II,III 2) I, II only 3) I,III only 4) III only
252
SR.INTER - IIT ADVANCED - VOL - 2 252
COMPLEX COMPOUNDS JEE-ADV CHEM-VOL-II
SR-MAIN-CHEM-VOL-II
2. Which of the following facts about the complex
27. In Fe  CO 5 the Fe  C bond possess
1)   character only 2)  character only Cr  NH 3   Cl3 is wrong. [AIEEE-2011]
 6
3) ionic character only4) Both  &  character 1) The complex innvolves d 2 sp3 hybridisation and
LEVEL-III - KEY is octahedral shape
1) 2 2) 3 3) 2 4) 2 5) 1 6) 4 7) 1 2) The complex is paramagnetic
3) The complex is an outer orbital complex
8) 2 9) 4 10) 1 11) 3 12) 3 13) 3 14) 2
4) The complex gives white precipitate which silver
15) 3 16) 4 17) 2 18) 3 19) 4 20) 4 21) 4 nitrate solution.
22) 1 23) 1 24) 1 25) 1 26) 1 27) 4 3. The magnetic moment (spin only) of

LEVEL-III - HINTS  NiCl4 2 [AIEEE-2011]


2. Primary valecy corresponds to oxidation s t a t e 1) 1.8BM 2) 5.86 BM
and secondary valecy corresponds to coordination 3) 2.82 BM 4) 3.82 BM
number primary=3, secondary=6. 4. Which among the following will be named as
dibromidobis (ethylene diamine) chromium III
3. Formula of complex is  MCl2  H 2O  4  Cl .
bromide [AIEEE - 2012]
Ionisable Cl  atoms are only one. M1V1  M 2V2   
4. IV of four complexes 1) CrBr2  en 2  2) CrBr2  en 2 
 
(1) is 3, (2) is 2, (3) is 1, (4) is 0
 2
6.  NO and  SCN are ambidentate ligands which have
2
 3) CrBr2  en   4) CrBr2  en  
two donor atoms. 5. Which of the following complex species is not
 expected to exhibit optical isomerisim
9. The complex Co(en) 2 Cl2  shows geometrical
[MAINS-2013]
as well as optical isomerism. 
1) Co  en  2 Cl2  2) Co  NH 3 3 Cl3 
15.   n  n  2  where n =5.  3
3) Co  en   NH 3 2 Cl2  4) Co  en 3 
17. For Mn 2  n  5 For Co 2  n  3
For Fe 2  n  4 PREVIOUS AIEEE QUESTIONS - KEY
18. For Fe 3 and Mn 2  n  5 1) 3 2) 3 3) 3 4) 1 5) 2
For Co 3  n  4 For Cr 3  n  3
LEVEL-IV
19. Cl is weak field ligand and ( N  is strong field
2
ligand so, in  NiCl4  hybridisation is sp3 in 1. The coordination numbers of Co and Al in
2 [CoCl (en) 2 ]Cl and k3 [ Al (C2O4 )3 ], respectively,,
 Ni  CN  4  hybridisation is dsp 2 .
are (en=ethane-1, 2-diamine)
PREVIOUS (JEE MAINS) (2019 Main, 12 April II)
QUESTIONS 1) 5 and 3 2) 3 and 3 3) 6 and 6 4) 5 and 6
2. The species that can have a trans-isomer is
1. Which one of the following has an optical
(en = ethane -1, 2-diamine, ox= oxalate)
isomer [AIEEE-2010]
(2019 Main, 10 April I)
2 2
1)  Zn  en 2 
  2 
2)  Zn en  NH 3   1) [ Pt (en)Cl2 ] 2) [Cr (en) 2 (ox )]

3 3 3) [ Pt (en) 2 Cl2 ]2 4) [ Zn(en)Cl2 ]


3) Co  en 3  4) Co  H 2O 4 en 

253
253 SR.INTER - IIT ADVANCED - VOL - 2
COMPLEX COMPOUNDS JEE-ADV CHEM-VOL-II
SR-MAIN-CHEM-VOL-II
3. The maximum possible denticities of a ligand 5. The following ligand is (2019Main, 8 April I)
given below towards a common transition and
inner-transition metal ion, respectively, are
(2019 Main, 9 April II)

1) 8 and 8 2) 8 and 6 1) hexadenate 2) tetradentate


3) 6 and 6 4) 6 and 8 3) bidenate 4) tridenate
4. The one that will show optical activity is (en = 6. The total number of isomers for a square pla-
ethane-1, 2-diamine) (2019 Main, 9 April I) nar complex [ M ( F )(Cl )( SCN )( NO2 )] is
(2019 Main, 10 Jan I)
1) 12 2) 16 3) 4 4) 8
7. The compound used in the treatment of lead
1) poisoning is(2019 Main, 12 April II)
1) D-penicillamine 2) dessferrioxime-B
3) cis-platin 4) EDTA
8. Complete removal of both the axial ligands
(along the z-axis) from an octahedral complex
leads to which of the following splitting patterns?
(relative orbital energies not on scale).
2) .
(2019 Main, 12 April I)

1) 2)
3)

4) 3) 4)

254
SR.INTER - IIT ADVANCED - VOL - 2 254
COMPLEX COMPOUNDS JEE-ADV CHEM-VOL-II
SR-MAIN-CHEM-VOL-II
9. The complex ion that will lose its crystal field 14. The calculated spin only magnetic moments
stabilisation energy upon oxidation of its metal (BM) of the anionic and cationic species of
to + 3 state us [ Fe( H 2O)6 ]2 and [ Fe(CN )6 ], respectively are
(2019 Main, 8 April II)
1) 0 and 4.9 2) 284 and 5.92
3) 0 and 5.92 4) 4.9 and 0
15. The compound that inhibits the growth of
tumors is (2019 Main, 8 April II)

Ignore pairing energy 1) trans- [ Pt (Cl ) 2 ( NH 3 ) 2 ]


(2019 Main, 12 April I) 2) cis- [ Pd (Cl )2 ( NH 3 ) 2 ]
2 2
1) [Co( phen)3 ] 2) [ Ni ( phen)3 ] 3) cis- [ Pt (Cl ) 2 ( NH 3 ) 2 ]
3) [ Zn( phen)3 ]2 4) [ Fe( phen)3 ]2 4) trans- [ Pd (Cl )2 ( NH 3 ) 2 ]
10. The crystal field stabilisation energy (CFSE) 16. The correct order of spin only magnetic
of [ Fe( H 2O)6 ]Cl2 and K 2 [ NiCl4 ], respec moment of metal ions in the following low
tively, are (2019 Main, 10 April II) spin complexs,
[V (CN ) 6 ]4 ,[ Fe(CN ) 6 ]4 ,[ Ru ( NH 3 ) 6 ]3 , and [Cr ( NH 3 ) 6 ]2  ,
1) 0.4 0 and  1.2t 2) 0.4 0 and  0.8 t
is
3) 2.4 0 and  1.2t 4) 0.6 0 and  0.8 t (2019 Main, 8 April I)
11. The incorrect statement is 1) Cr 2  Ru 3  Fe 2  V 2
(2019 Main, 10April II)
2) V 2   Cr 2  Ru 3  Fe 2
1) the gemstone, ruby, has Cr 3 ions occupy
3) V 2   Ru 3  Cr 2   Fe 2
ing the octahedral sites of beryl
4) Cr 2  V 2  Ru 3  Fe 2
2) the color of [CoCl ( NH 3 )5 ]2  is violet as it
17. The magnetic moment of an octahedral
absorbs the yellow light homoleptic Mn(II) complex is 5.9 BM. The
3) the spin only magnetic moments of suitable ligand for this complex is
Fe( H 2O ) 6 ]2  and [Cr ( H 2O ) 6 ]2  are nearly 1) CN  2) ethylenediamine
similar 3) NCS  4) CO
4) the spin only magnetic moment of
18. The pair of metal ions that can given a
[ Ni ( NH 3 ) 4 ( H 2O ) 2 ]2 is 2.83 spin-only magnetic moment of 3.9 BM for the
12. Three complexes, complex [ M ( H 2O)6 ]Cl2 , is(2019 Main, 12
2 3
[CoCl ( NH 3 )5 ] (I), [Co(NH3 )5 H 2O] (II) and Jan I)
[Co(NH 3 )6 ]3 ( III ) 1) Co 2 and Fe 2 2) Cr 2  and Mn 2
absorb light in the visible region. The correct 3) V 2 and Co 2 4) V 2 and Fe 2
order of the wavelength of light absorbed by 19. The matal d-orbitals that are directly facing
them is (2019 Main, 10 April I) the ligands in K 3 [Co(CN )6 ] are
1) II > I > III 2) I > II > III (2019 Main, 12 Jan I)
3) III > I > II 4) III > II > I
1) d xz , d yz and d z 2 2) d x2  y 2 and d z 2
13. The degenerate orbitals of [Cr ( H 2O ) 6 ]3 are
(2019 Main, 9 April I) 3) d xy ,d xz and d yz 4) d x z a n d d x 2  y 2
1) d z 2 and d xz 2) d xz and d yz

3) d x2  y 2 and d xy 4) d yz and d z 2
255
255 SR.INTER - IIT ADVANCED - VOL - 2
COMPLEX COMPOUNDS JEE-ADV CHEM-VOL-II
SR-MAIN-CHEM-VOL-II
20. Mn2 (CO )10 is an organometallic compound 26. Wilkinson catalyst is
due to the presence of (2019 Main, 12 Jan I) (2019 Main, 10 Jan I)
1) Mn  C bond 2) Mn  O bond 1) [(Et3 P)3 RhCl ]
3) C  O bond 4) Mn  Mn 2) [( Et3 P)3 IrCl ]( Et  C2 H 5 )
21. The number of bridging CO ligand(s) and
3) [(Ph3 P)3 RhCl ] 4) [(Ph3 P)3 IrCl ]
Co-Co bond(s) in Co2 (Co)8 , respectively are
27. Homoleptic octahedral complexes of a metal
(2019 Main, 11 Jan II)
1) 2 and 0 2) 0 and 2 ion ' M 3 ' with three monodentate ligands
3) 4 and 0 4) 2 and 1 L1 , L2 and L3 absorb wavelengths in the region
22. The coordination number of Th in of green, blue and red respectively. The increas-
K 4 [Th(C2O4 ) 4 (OH 2 ) 2 ] is (C2O42   Oxalato) ing order of the ligand strength is
(2019 Main, 11 Jan II) (2019 Main, 9 Jan II)
1) 14 2) 10 1) L1  L2  L3 2) L2  L1  L3
3) 8 4) 6
3) L3  L1  L2 4) L3  L2  L1
23. Match the metals (Column I) with the
coordination compound(s)/enzyme(s) (Column 28. The complex, that has highest crystal field
II). splitting energy    ,is (2019 Main, 9 Jan II)
(2019 Main, 11 Jan I)
1) [Co( NH 3 )5 Cl ]Cl2
Column I Column II
2)  C o  N H 3 5  H 2 O   C l 3
(A) Co (i) Wilkinson catalyst
(B) Zn (ii) Chlorophyll 3) K 3 [Co(CN )6 ]
(C) Rh (iii) Vitamin B12
4) K 2 [CoCl4 ]
(D) Mg (iv) Carbonic anhydrase
29. The highest value of the calculated spin only
(A) (B) (C) (D) magnetic moment (in Bm) among all the
(1) (i) (ii) (iii) (iv) transition metal complexes is
(2) (iv) (iii) (i) (ii) (2019 Main, 9 Jan I)
(3) (iii) (iv) (i) (ii) 1) 5.92 2) 3.87
(4) (ii) (i) (iv) (iii) 3) 6.93 4) 4.90
24. The difference in the number of unpaired 30. Two complexes
electrons of a metal ion in its high-spin and low-
[Cr ( H 2O)6 ]Cl3 ( A)and [Cr ( NH 3 )6 ]Cl3 (B) are
spin octahedral complexes is two. The metal ion
is (2019 Main, 10 Jan II) violet and yellow coloured, respectively. The
incorrect statement regarding them is
1) Mn 2 2) Fe2 
(2019 Main, 9 Jan 1)
3) Ni 2 4) Co 2
1)  0 value for (A) is less than that of (B)
25. A reaction of cobalt (III) chloride and ethyl-
2) both absorb energies corresponding to their
ene diamine in a 1:2 mole ratio generates two
complementary colours
isomeric products A (violet coloured) and B
(green coloured). A can show optical activity, 3)  0 values of (A) and (B) are calculated
but B is optically inactive. What type of isomers from the energies of violet and yellow light,
does A and B represent ? respectively
(2019 Main, 10 Jan II) 4) both are paramagnetiv with three unpaired
1) Ionisation isomers 2) Coordination isomers electrons
3) Geometrical isomers 4) Linkage isomers

256
SR.INTER - IIT ADVANCED - VOL - 2 256
COMPLEX COMPOUNDS JEE-ADV CHEM-VOL-II
SR-MAIN-CHEM-VOL-II

31. The IUPAC name of the complex 36. Complex X of composition Cr(H2O)6 Cln has a
spin only magnetic moment of 3.83
[Pt(NH3)2Cl(NH2CH3)]Cl is :
BM. It reacts with AgNO3 and shows geo-
1) Diammine (methanamine) chlorido metrical isomerism. the IUPAC nomencla
platinum (II) chloride ture of X is :
2) Bisammine (methanamine) chlorido 1) Tetraaquadichlorido chromium (III) Chloride
platinum (II) chloride dihydrate
2) Hexaaqua chromium (III) chloride
3) Diamminechlorido (aminomethane) 3) Dichloridotetraaqua chromium (IV) chloride di-
platinum(II) chloride hydrate
4) Diamminechlorido (methanamine) 4) Tetraaquadichlorido chromium (IV) chloride di-
hydrate
platinum (II) chloride
37. The correct order of the spin - only magnetic
32. The theory that can completely/properly moments of the following complexes is :
explain the nature of bonding in (I) [Cr(H2O)6]Br2
[Ni(CO)4] is : (II) Na4[Fe(CN)6]
1) Werner’s theory 2)
(III) Na3[Fe(C2O4)3] (  0  P )
Crystal field theory3) Valence bond theory
4) Molecular orbitaltheory (IV) (Et4N)2[CoCl4]
33. Among the statements(a)-(d), the incorrect (1) (III) > (I) > (II) > (IV)
ones are- (2) (I) > (IV) > (III) > (II)
(a) Octahedral Co(III) complexes with strong (3) (II)  (I) > (IV) > (III)
field ligands have very high magnetic moments (4) (III) > (I) > (IV) > (II)
(b) When D0 < P, the d-electron configuration 38. The isomer(s) of [Co(NH 0
) Cl2] that has/have
3 4
4
of Co(I I I ) in an octahedr al complex is t eg eg2 a Cl-Co-Cl angle of 90 , is /are:
(c) Wavelength of light absorbed by [Co(en)3]3+ 1) meridional and trans 2) cis and trans
is lower than that of [CoF6]3– 3) trans only 4) cis only
(d) If the D0 for an octahedral complex of NUMERICAL VALUE QUESTION
Co(III) is 18,000 cm–1, the Dt for its tetrahedral 39. Complexes (ML5) of metals Ni and Fe have
complex with the same ligand will be 16,000 cm–1 ideal square pyramidal and trigonal
(1) (a) and (b) only (2) (c) and (d) only bipyramidal g r o m e t r i e s ,
(3) (b) and (c) only (4) (a) and (d) only respectively.The sum of the 90°, 120° and 180°
34. The complex that can show fac-and mer-iso- L-M L angles in the two complexes
mers is : is __________ .
(1) [Pt(NH3)2Cl2] (2) [Co(NH3)4Cl2]+ Ans. (20 to 20)
(3) [Co(NH3)3(NO2)3] (4) [CoCl2(en)2]
35. [Pd(F) (Cl) (Br) (I)]2- has n number of geometri-
cal isomers. Then , the spin - only magnetic LEVEL-IV - KEY
moment and crystal field stabilisation energy 1. 4 2. 3 3. 4 4. 1 5. 2 6. 1 7. 4
[CFSE] OF [Fe(CN)6]n-6, respectively, are: 8. 2 9. 4 10. 2 11. 1 12. 2 13. 2 14. 1
[Note : Ignore the pairing energy]
15. 3 16. 2 17. 3 18. 3 19. 2 20.1 21. 4
1) 2.84 BM and -1.6  0
22. 2 23. 3 24. 4 25. 3 26. 3 27. 3 28. 3
2) 1.73 BM and -2.0  0
29. 1 30. 3 31. 4 32. 4 33. 4 34. 3 35. 2
3) 0 BM and -2.4  0 36. 1 37. 2 38. 4
4) 5.92 BM and 0

257
257 SR.INTER - IIT ADVANCED - VOL - 2
COMPLEX COMPOUNDS JEE-ADV CHEM-VOL-II
SR-MAIN-CHEM-VOL-II
LEVEL-IV - HINTS (C) D-penicillamine is used for copper
2
1. en and C2O 4 are a bidentate ligand. So poisoning
coordination number of [Co(Cl)(en)2]Cl is 5 and (D) desferrioxime B is used for iron poi-
K3[Al(C2O4)3] is 6 soning
8. Ligand filed exert mass repulsion along
x, y axis as compared to Z-axis so d x2  y 2 and dxx
2. will have increase in energy y

3. General coordination number of CN– in 9.


transition element is 6 and in inner transition
element is 8-12. because inner transition metal
10.
ions can make available more number of
11. In gemstone, ruby has Cr3+ ion occupy-
vacant orbitals of nearly same energy than
ing the octahedral sites of aluminium oxide (Al2O3)
transition metal ions. The high effective normally occupied by Al3+ion.
nuclear charge of inner-transition metal ions 12.
make them form complex with high coordina
tion number.

4.

13.
5. Both nitrogen & oxygen are donating
atoms.
6.

7. (A) EDTA (ethylene diamine tetra acetate)


14.
is used for lead poisoning
(B) Cis platin is used as a anti cancer drug

258
SR.INTER - IIT ADVANCED - VOL - 2 258
COMPLEX COMPOUNDS JEE-ADV CHEM-VOL-II
SR-MAIN-CHEM-VOL-II
least one chemical bond between carbon and a
metal.

21.
15.cis–[PtCl2(NH3)2] is used in chemotherapy to in-
hibits the growth of tumors
16.
22. Th is a metal having large size and ox-
alate is a bidentate ligand hence its co-ordination
number in given complex is 10.

23.
17. The magnetic moment of the magnitude
5.9 BM suggest the presence of 5 unpaired
electrons inMn (II).This can be cross
verified by putting the value (5) of unpaired
electrons in the formula, 24.
  n(n  2) BM
Thus, the valence electronic configuration 25. Cobalt (III) chloride on reaction with
of Mn(II) in the complex is ethylenediemine in ratio 1:22 isomeric products
complexes A and B

The octahedral homoleptic complex sug-


gests sp3d2 - hybridisation in the complex , i.e.

26. Wilkinson’s catalyst is a   bonded or--


Thus , 5 unpaired electrons are present
in the complex which suggest the presence of a ganometallic compound [Ph3P)3RhCl].It is commer-
weak lig andlike NCS - cially used for hydrogenation of alkenes and veg-
18. etable oils (unsturated).
IUPAC name chloridotris (triphenylphosphene)
rhodium(I).
27.

19. Given K3[Co(CN)6] is inner orbital com-


plex with hybridization d2sp3 and octahedral geom-
etry. Ligands are approaching metal
along the axes. Hence, d x2  y 2 , d z 2 orbitals are di-
rectly in front of the ligands.
20. Organometalic compound contains at 28. As CN - is a strong field ligand.
K3[Co(CN)6] will have maximum '  '

259
259 SR.INTER - IIT ADVANCED - VOL - 2
COMPLEX COMPOUNDS JEE-ADV CHEM-VOL-II
SR-MAIN-CHEM-VOL-II
29. Maximum number of unpaired electron of
metal or metal ion in complexes = n = 5
 s  n  n+2   35  5.916  5.92
30.  0 is calculated from the energies of absorbed
radiation not from emitted radiation (complemen-
tary colour).
31. Conceptual.
32.In complex [Ni(CO) 4] decrease in Ni–C bond 35. [Pb (F) (Cl) (Br) (I)]2- have three geo-
metrical isomer so formula for [Fe(CN)6]n-6 is
length and increase in C–O bond length as well as
[Fe(CN)6]3- and CFSE for this
it’s magnetic property is explained by MOT.
complex is Fe3  3d 5 4 s 0 +
33.

36. Cr(H2O)6 Cln


if magnetic mement is 3.83 BM then it
containthree unpaired electrons. It means
(c) Splitting power of ethylenediamine (en) is chromiumin +3 oxidation state so molecu-
greater than fluoride (F–) ligand therefore more lar formula isCr(H2O)6Cl3
energy absorbed by [Co(en)3]3+ as compared to This formula have following isomers
[CoF 6] 3–. (a) [Cr(H2O)6]Cl3 : react with AgNO3 but
So wave length of light absorbed by [Co(en)3]3+ doesnot show geometrical isomerism.
is lower than that of [CoF6]3– (b) [Cr(H2O)5Cl]Cl2.H2O react with
4 AgNO3 but does not show geometrical isomerism.
t  0 (c) [Cr(H2O)4Cl2]Cl.2H2O react with
9 AgNO3 & show geometrical isomerism.
so if  0  18, 000 cm 1 (d) [Cr(H2O)3Cl3].3H2O does not react
(d) with AgNO3 & show geometrical isomerism.
4
 t  18000  8000 cm 1
9 [Cr(H2O)4Cl2]Cl.2H2O react with AgNO3
Statement (a) and (d) are incorrect. & show geometrical isomerism and it’s IUPAC
nomenclature is Tetraaquadichlorido
34. [Ma3b3] type complex shows fac and chromium (III) Chloride dihydrate.
mer isomerism.

260
SR.INTER - IIT ADVANCED - VOL - 2 260
COMPLEX COMPOUNDS JEE-ADV CHEM-VOL-II
SR-MAIN-CHEM-VOL-II
37.

Unpaired electrons = 3
Magnetic moment = 15BM
= 3.87 BM
Hence order of magnetic monent is I > IV > III >
II

38.

cis isomer has Cl-Co-Cl angle of 900


39. Conceptual.

261
261 SR.INTER - IIT ADVANCED - VOL - 2
COMPLEX COMPOUNDS JEE-ADV CHEM-VOL-II
SR-MAIN-CHEM-VOL-II
ADVANCED POINTS  NH 3  C5H5 N  en  SO32
12. Factors influencing the magnitude of  o :
 NH 2OH  NO 2  Phen  CH3
I. Nature of the metal cation:
a) Among the cations from atoms of the same  CN   CO
transition series cations with a higher oxidation This order independent of the nature of the central
states has a larger value of  o than that of lower metal ion and the geometry of the complex.
III. Geometry of the complex:
oxidation states, because the central ion with higher
As the geometry changes  value chages.
oxidation state will polarise the ligands more
effectively and thus the ligands would approach such 4 4
cation more closely. sp  o   t ;  sp   o ;  t   o
3 9
2 
Ex:  0 for  Fe II  H 2 O 6   10, 400 cm 1  3d 6  13. ORGANO METALLIC COMPOUNDS
EXAMPLES
3 a) Grignard Reagent R –Mg –X where R is a alkyl or
 0 for  Fe  H 2 O 6   13, 700 cm  3d 
III 1 5
aryl group and X is halogen
b) Among the cations having same number of ‘d’ b)  CH 3  Sn ,  C 2 H 5  Pb, Al 2  CH 3  Al 2  C 2 H 5  etc
4 4 6 6
electrons and the same geometry of the complex, Bridging groups
cations with higher oxidation state has large value
HC CH CH
of  o . 3 3 3

Al Al
2
Ex:  0 for  V II  H 2 O 6   12, 400 cm 1  3d 3  H3C CH3 CH3
3 ii) pie (p) bonded organometallic compounds : These
 0 for  Cr III  H 2 O 6   17, 400 cm 1  3d 3  are the compounds of metal with alkenes, alkynes,
c) In case of complexes having the cations with the benzene and other ring compounds.
same charges but with different number of ‘d’
electrons in the central metal cation, the magnitude e.g. Zeise’s salt  
K  PtCl3 2  C 2 H 4 
 

of  o decreases with the increase of the number of H H

‘d’ electrons. C

Ex: K+
Cl C
2
 0 for CO II  H 2 O 6   9, 300 cm 1  3d 7  Pt H H
Cl Cl
2
 0 for  Ni II  H 2 O 6   8,500 cm 1  3d 8 
Potassium trichloro
d) As the quantum number of the ‘d’ orbitals of the 1

central atom increases  o value increases. ( -ethylene)platinate(II)


2

3 Fe +2
Ex:  0 for  CO III  NH 3 6   23, 000 cm 1  3d 6  Has both  and  ligands.

 0 for  Rh III  NH 3 6 
3
 34, 000 cm 1  4d 6  Ferrocene 
Fe 5  C5 H 5  2
1

II. Strong/weak ligands:


Strong ligands possess higher  o values. bis ( 5 -cyclopentadienyl)iron(II)
Spectrochemical series: Sandwich complex
   
I  Br  Cl  SCN  N 3 bis (benzene) chromium (0), Cr

  C2 H5O 2 PS2  F   NH 2  2 CO 
Cr 6  C6 H 6  2

sandwich complex
 OH   C2O 24  H 2O  NCS  H 
iii)  &  bonded organometallic comp.
 CN   NH 2CH 2COO  eg Metal carbonyls

262
SR.INTER - IIT ADVANCED - VOL - 2 262
COMPLEX COMPOUNDS JEE-ADV CHEM-VOL-II
SR-MAIN-CHEM-VOL-II
14. Metal carbonyls: e 0  
 Mn  CO 6    Mn  CO 6 
 The highest occupied molecular orbital of carbon
 EAN  37   EAN  36 
monoxide, the  2p Z orbital, holds two electrons. This
0
electron pair is loosely held and hence is available 2)  V  CO 6  can act as oxidising agent. The
for donation to metal. The bonding takes place complex gains an electron to attain the noble gas
when the filled  2p Z molecular orbital of CO configuration and hece obey Sidgwick EAN rule.
overlaps with an empty orbital of the metal. As a e0  
 V  CO 6    V  CO 6 
result, a normal ligand - to - metal  bond is formed.  EAN  35   EAN 36 
 CO also has two degenerate, empty, anti - bodning 0
 orbitals. These are the lowest unoccupied 3)  Mn  CO 5  undergoes dimerisation to attain the
molecular orbitsls of CO, mutually perpendicular noble gas configuration and hence obey Sidgwick
to each other. The overlap of one of these vacant EAN rule.
anti - bonding molecular orbitals with a filled metal 0
orbital of  - symmetry results in the formation of 2  Mn  CO 5    Mn 2  CO 10 
 EAN 35   EAN  36 
a second bond called dative bond. The dative bond
serves to pass the excessive negative charge(that  Anionic carbonyl complexes are called carbonylate
ions, these are also referred to as carbometallates.
accumulates on the metal due to the formation of
Ex:
  bond) to the ligand. 2 
 Back donation of electrons from the metal to the 1)  Ti  CO 6  ,  V  CO 6  : Isoelectronic,
ligands is possible only if the metal is in a low
oxidation state. Hence, CO ligand stabilizes low isostructural with  Cr  CO 6  .
oxidation states of metals in its complexes. 3 2
2)  V  CO 5  ,  Cr  CO 5  ,
 The   and   bonding mutually supplement each
other. The donation of electrons to metal, during 
 Mn  CO 5  : Isoelctronic, isostructural with
the formation of  bond, increases the partial
positive charge on the CO ligand, making it a better  Fe  CO 5 
acceptor. The back donation of metal electrons to 4 3
the ligands increases the partial negative charge on 3) Cr  CO 4  ,  Mn  CO 4  ,
CO, making it, in effect, a better donor. This mutual 2 
reinforcement is called synergism. As a result of  Fe  CO  4  , Co  CO 4  : Isoelectronic,
synergism,   bonding strengthens   bonding
isostructural with  Ni  CO  4 
vice versa.
 A consequence of synergism is that CO ligand forms  As the negative charge on the coordination sphere
a large number of complexes with transition metals of a carbonyl complex increases, the extent of back
in low oxidation states, even though it is a poor bonding also increases. This, in turn, leads to an
Lewis base for the other species. Another increase in metal - carbon bond order and a
consequence of synergism is that it increases the decrease in carbon - oxygen bond order.
bond order of Ex: In
metal - carbon bond and decreases the bond order
2 2 
of carbon - oxygen bond. This results in  V  CO 5  , Cr  CO 5  ,  Mn  CO 5 
a) shortening of metal - carbon bond as compared I  II   III 
to the metal - carbon single bond.
Carbon - oxygen bond order: I < II < III
b) lengthening of carbon - oxygen bond as
compared to carbon - oxygen bond. Carbon - oxygen bond length: I > II > III
 Metal carbonyls exhibit a strong tendency to achieve Carbon - oxygen bond strength: I < II < III
Sidgwick EAN values and as a result Metal - carbon bond strength: I > II > III
0
1)  Mn  CO 6  can act as a reducing agent. The
complex loses an electron to attain the noble gas
configuration and hence obey Sidgwick EAN rule.
263
263 SR.INTER - IIT ADVANCED - VOL - 2
COMPLEX COMPOUNDS JEE-ADV CHEM-VOL-II
SR-MAIN-CHEM-VOL-II
15. Factors affecting the stability of Ex: When ethylene diamine(en) is allowed to react with
complexes: 2
hydrated complex ion,  M  H 2 O 6  , it replaces
I. Charge and size of the metal ion:
In general the metal ion with larger charge and two H 2 O molecules from it, since it is a bidentate
smaller size form more stable complexes. ligand.
3 4 This process increases the number of particles in
Stability of:  Fe  CN 6    Fe  CN 6 
    the system and hence its disorder and entropy.
 log  31  log 8.3 2
II. Irving - Willium order: Thus the complex  M  H 2 O  4  en   is more
Stabilities of high spin complexes of the ions 2
stable than  M  H 2 O 6 
between Mn 2  and Zn 2 with a given ligand vary
in the order: 2
 M  H 2 O 6   en 
Mn 2  Fe 2   Co 2   Ni 2   Cu 2  Zn 2 2
Radii of these ions are in the order:  M  H 2 O 4  en    2H 2 O
Mn 2  0.91A0   Fe 2  0.83A0  VII. Chelate ring size:
 The large the number of the chelate rings in a
 Co 2  0.82A0   Ni 2   0.78A0  complex, the greater is its stability.
 The stability of the complexes also depends on the
 Cu 2  0.69A0   Zn 2   0.74A  0
number of the atoms present in the rign.
III. Electronegativity of the central metal ion:  The chelates containing 3 - membered ring including
A cental metal ion with higher electronegativity would the metal are very unstable.
give the most stable complexes.  4- membered chelate rings are rare and occur in
IV. Size and charge of ligand: carbonate, nitrate and sulphate chelates.
 5-membered chelates are frequently more stable
For charged ligands, the higher the charge carried
than the 6-membered chelates when the atoms in
by them and the smaller their size, the more stable
the rign are joined by single bonds only.
are the complexes formed
 6-membered chelates are more stable than
(for class a metals)
5-membered chelates of heterocyclic ligands or of
(Class a metals: Sc, Ti, V, Cr etc) ligands involving conjugation in the chelate ring.
2 2
 FeF   FeCl VIII. Steric effects:
Ex: Stability of When a bulky group is either attached to or present
 log 10   log  20 
6

near the donor atoms of a ligand, mutual repulsion


V. Basic character of the ligands:
among the ligand occurs and consequently the metal
The more basic is the ligand, more easily it can
- ligand bond is weakened. Thus large bulky ligands
donate electron pairs to the central ion and hence form less stable complexes than do the analogous
more easily it can form complexes of greater stability. smaller ligands.
VI. Chelating effect:
 Complexes containing chelate rings are usually more
stable than similar complexes containing no rings,i.e.,
multidentate ligands ingenral form more stable
complexes than the monodentate ligands.
2 2
 Ni  en 3    Ni  NH 3 6 
Ex: Stability:   aq   aq 
 log 18.1  log  7.99 
 The greater the amount of disorder produced in
the products during the reaction relative to the
reactants, the greater the increase in entropy during
the reaction and hence the greater the stability of
the products.
264
SR.INTER - IIT ADVANCED - VOL - 2 264

You might also like